Сохранен 524
https://2ch.hk/sci/res/198318.html
24 декабря Архивач восстановлен после серьёзной аварии. К сожалению, значительная часть сохранённых изображений и видео была потеряна. Подробности случившегося. Мы призываем всех неравнодушных помочь нам с восстановлением утраченного контента!

Математика, тред № 2

 Аноним Срд 21 Май 2014 22:47:27  #1 №198318 
1400698047728.png

Снова начинаю с задачки.

1. Имеется ящик с 1000 одинаковыми по внешнему виду камнями. Из них ровно один радиоактивен. У вас есть детектор, который за 1 рубль возьмёт у вас на проверку любое количество камней и сообщит, есть ли среди них радиоактивный. При этом, если в группе был радиоактивный камень, все камни из этой группы станут радиоактивными. За каждый нерадиоактивный камень (вы должны гарантировать отсутствие радиоактивности) заказчик платит вам 1 рубль. Как обеспечить максимальную выручку? Какой она будет?

Предыдущий тред: http://2ch.hk/sci/res/187741.html

Аноним Чтв 22 Май 2014 01:22:20  #2 №198338 

>>198318
Не вижу условия, по которому нельзя перебрать камни по одному.

Аноним Чтв 22 Май 2014 05:12:30  #3 №198350 

>>198318
Нужно уточнить, что максимизируется - гарантированная выручка, математическое ожидание выручки или что-то иное? 955

Аноним Чтв 22 Май 2014 05:15:11  #4 №198351 

>>198338
Тогда ты с вероятность 999/1000 потратишь 999 рублей на проверку, и получишь за них те же 999 рублей от заказчика.

Я считаю надо половину (500) камней проверить за 1 рубль, если там был радиоактивный - продавать оставшуюся половину за 500. Если проверенные чистые - продавать их и половину от оставшихся проверять, и так пока не закончатся. С вероятностью 100% прибыль будет >=499 р.

Аноним Чтв 22 Май 2014 06:11:33  #5 №198356 

>>198351
Почему именно половину? Можно, например, в золотом сечении делить или еще как. Хотя с половиной проще, конечно.

Аноним Чтв 22 Май 2014 08:16:08  #6 №198369 

>>198351
В таком случае, выгоднее разделить камни на группы по 100. С каждой группы ты получишь по 99 рублей. Всего будет 9 групп (в одной из групп будет радиоактивный камень). 9*99=891р выручки. Может есть даже еще более оптимальное разбиение.

Аноним Чтв 22 Май 2014 10:24:52  #7 №198390 

>>198369
Для разбиения A камней на n равных групп оптимально sqrt(A) групп по sqrt(A) камней. Т.е. 32 группы по 31-32 камня

Аноним Чтв 22 Май 2014 10:31:01  #8 №198393 

>>198318
Х - количество камней в группе
Цена проверки камней - 1000 деленное на Х
Прибыль от продажи - 1000 минус Х
Тебе нужно маскимизировать функцию f(X) = 1000 - Х - 1000/Х
Ее производная: 1000/X^2 - 1 равна нулю в точке X = 100
Знак производной сменяется с "+" на "-", следовательно эта точка максимум.

Аноним Чтв 22 Май 2014 10:39:58  #9 №198394 

>>198390
это правда если все группы проверять, на самом деле достаточно половину в среднем. Тогда количество групп увеличивается до sqrt(2A): 44 группы по 22-23 камня. Средняя выручка ~= 1000 - 22 - 23 = 955

Аноним Чтв 22 Май 2014 10:42:34  #10 №198395 
1400740954242.jpg

>>198393
>Ее производная: 1000/X^2 - 1 равна нулю в точке X = 100
в голос с этого арифметика

Аноним Чтв 22 Май 2014 13:40:17  #11 №198413 

Гарантированная 955, средняя 957.329 (это точное значение).

Аноним Чтв 22 Май 2014 13:54:03  #12 №198416 

Максимальная выручка - 998р. Как получить: угадать, какой камень радиоактивен и проверить его.

Аноним Чтв 22 Май 2014 14:05:49  #13 №198418 

>>198416
Но если ты угадал, зачем тебе его проверять?

Аноним Чтв 22 Май 2014 14:09:03  #14 №198420 

>>198413
Я совершаю глупы ошибки, гарантированная будет 954.

Аноним Чтв 22 Май 2014 14:11:42  #15 №198421 

>>198418
Он всё правильно написал. Иначе нельзя гарантировать отсутствие радиоактивных камней.

Аноним Чтв 22 Май 2014 14:56:24  #16 №198432 

>>198413
>Гарантированная 955
Как?
>средняя 957.329 (это точное значение)
Средняя в твоём алгоритме, или в лучшем?

Аноним Чтв 22 Май 2014 15:05:05  #17 №198435 

>>198432
>Как?
Никак, я там ошибся, на самом деле 954. Нужно проверить вначале 45, дальше, если не найден радиоактивный, 44, ... и т.д. пока на очередном шаге не придётся проверить все оставшиеся.
>Средняя в твоём алгоритме, или в лучшем?
Лучшем.

Аноним Чтв 22 Май 2014 15:07:07  #18 №198436 

>>198435
>Лучшем
Обоснуй. Численно можно и до 973 дойти (аналитически ещё не осилил)

Аноним Чтв 22 Май 2014 15:24:32  #19 №198439 
1400757872293.jpg

А зачем проверять? Продадим 999 камней, а 1 выкинем или продадим другому, чтобы покупатель подумал, что мы нашли радиоактивный, и все.

qsc !zySjv1ExpY Чтв 22 Май 2014 15:28:07  #20 №198440 

>>198318
давно логарифмы прошёл, дурень? нахуй иди.

Вот вам моя задачка.
Имеется ящик с 1000 одинаковыми по внешнему виду камнями. Из них ровно один радиоактивен. У вас есть детектор, который за 1 рубль возьмёт у вас на проверку любое количество камней и сообщит, есть ли среди них радиоактивный. При этом, если в группе был радиоактивный камень, все камни из этой группы станут радиоактивными. За каждый РАДИОАКТИВНЫЙ камень вам платят по рублю. Как увеличить выручку?

мимоктн

Аноним Чтв 22 Май 2014 15:30:59  #21 №198441 

>>198440
Единицу поделить на все.

Аноним Чтв 22 Май 2014 15:31:50  #22 №198442 

>>198440
>возьмёт любое количество камней
>если был радиоактивный камень, все камни группы станут радиоактивными
>За каждый РАДИОАКТИВНЫЙ камень вам платят по рублю. Как увеличить выручку?
Таки 1 рубль за проверку 1000 камней одной кучей, и 999 чистого профита после продажи.

>мимоктн
ты по ходу мимодбл

Аноним Чтв 22 Май 2014 16:00:22  #23 №198447 

>>198436
Ясно, что оптимальная стратегия для n>1 (технически, для 2 равноправна стратегия ничего не проверять) камней имеет вид:
1. проверить kn камней,
2. если в 1. нашёлся радиоактивный, то сдать оставшиеся, иначе применить оптимальную стратегию для n-kn.
Соответственно оптимальные средние выигрыши an и kn подчиняются соотношению
a1=0
an=-1+(n-kn)*(kn/n)+(an-kn+kn)*(1-kn/n)
kn - число от 1 до n-1, максимизирующее an.
Если это посчитать, то выйдет 957.329.

Задача из ОП-поста Аноним Чтв 22 Май 2014 16:09:16  #24 №198448 

>>198350
Конечно же гарантированная выручка, т.е. оптимальный алгоритм. В подобных задачах варианты с везением и различными исходами обычно неинтересен.

>>198420
>>198435
Да, всё верно. Можешь строго доказать? Также можно предлагаю обобщить задачу на случай n камней и k радиоактивных (или хотя бы 1000 камней и 2 радиоактивных).

Аноним Чтв 22 Май 2014 16:11:41  #25 №198449 

>>198447
>Если это посчитать
Как считал? Конкретнее, как выглядит формула kn?
Вот передо мной тысяча камней, сколько кидать в детектор и чему равно k1000?

Аноним Чтв 22 Май 2014 16:12:17  #26 №198450 

>>198448
>Также предлагаю
фикс

Аноним Чтв 22 Май 2014 16:13:37  #27 №198451 

>>198448
>В подобных задачах варианты с везением и различными исходами обычно неинтересен.
Как будто на дистанции (тысяча таких куч камней) нам нужно оптимизировать не матожидание. Но уточнение принято.

Аноним Чтв 22 Май 2014 16:24:07  #28 №198454 

Есть отрезок длиной 1 метр. С одного его конца на другой движется точка со скоростью 1 см в минуту. Каждую минуту отрезок "растягивается" на 5 см, т.е. длина отрезка увеличивается на 5 см и точка смещается относительно начала движения. Вопрос: за какое время точка достигнет другого конца отрезка?

Аноним Чтв 22 Май 2014 16:25:12  #29 №198455 

>>198454
И достигнет ли вообще?

Аноним Чтв 22 Май 2014 16:26:21  #30 №198456 

>>198449
Тривиальная программа же. k32=32, по крайней мере это одно из возможых значений.
>>198448
>Можешь строго доказать?
Могу, но лень. По существу, идея доказательства есть в >>198447, разница только в том, что оптимизируется гарантированное значение, а не среднее.

Аноним Чтв 22 Май 2014 16:27:23  #31 №198457 

>>198456
>k1000=32

Аноним Чтв 22 Май 2014 16:28:32  #32 №198458 

>>198456
Среднее значение может достигать 973.
(По поводу гарантированного вопросов не имею, доказывается при желании аналитически)

Аноним Чтв 22 Май 2014 16:30:21  #33 №198459 

>>198448
>Можешь строго доказать?
Я могу, а надо?

Аноним Чтв 22 Май 2014 16:31:40  #34 №198460 

>>198454
>. длина отрезка увеличивается на 5 см и точка смещается относительно начала движения
Как смещается? т.е. предполагаю что "пропорционально", но требую уточнения условия.

Аноним Чтв 22 Май 2014 16:40:37  #35 №198463 

>>198458
Не исключено, что у меня в программе ошибка. Можно было бы выложить программ для малого количества камней и разобраться, у кого ошибка. У меня выходит
a1=0
a2=0
a3=1/3
a4=1
a5=8/5
a6=7/3
a7=3
a8=15/4
a9=41/9

Аноним Чтв 22 Май 2014 16:45:42  #36 №198465 

>>198463
У меня по сути "подбор" исходя из предположения, что "хороший" вариант: имея N непроверенных камней проверить alpha*N, соответственно подбирается перебором alpha. Т.е. строгости не какой, но статистика для alpha = 0.015 - хорошая.

Аноним Чтв 22 Май 2014 16:54:23  #37 №198467 

>>198465
Всё же выложи результаты для малого количества камней. Теоретически, если в моих программе и доказательстве нет ошибок, то результаты моей программы должны быть >= результатов любой корректной стратегии. Но у тебя для 1000 выходит большее значение.

Аноним Чтв 22 Май 2014 16:55:26  #38 №198468 

>>198465
Да и ещё, в какую сторону ты округляешь?

Аноним Чтв 22 Май 2014 17:18:06  #39 №198469 

>>198465
Прогнал описанный тобой метод с округлением вверх - вышло 932.691.

Аноним Чтв 22 Май 2014 17:20:02  #40 №198470 

>>198460
Смещение такое же, как если ты возьмёшь резиновую ленту, поставишь на ней точку маркером и растянешь. Т.е. x(новое)=x(старое)+ ([длина ленты до рястяжения]/[длина ленты после растяжения]).

Аноним Чтв 22 Май 2014 17:22:06  #41 №198471 

>>198470
Обосрался с формулой, исправишь сам.

Аноним Чтв 22 Май 2014 17:25:47  #42 №198472 

>>198468
>Да и ещё, в какую сторону ты округляешь?
Округляю вверх.

Алсо, я хуй простой и проебался со сбросом генератора случайных чисел.

Для малого количества: для н=9 у меня тоже 41/9 , только это не "средний выигрыш" а средний "проёб" от 9. Ты это и имел в ввиду?

Аноним Чтв 22 Май 2014 17:39:39  #43 №198474 

>>198472
> Ты это и имел в ввиду?
Нет.
>для н=9 у меня тоже 41/9 , только это не "средний выигрыш" а средний "проёб" от 9
Если следовать твоему методу, то твоя стратегия до 9 - проверять по 1 камню. Выигрыш на самом деле выйдет 28/9: с вероятностью 1/9 будет проверен лишь 1 камень и выигрыш будет 7, с вероятностью 1/9 будет проверен лишь 1 камень и выигрыш будет 6,... . Несложно видеть, что в среднем выходит 28/9.

Аноним Чтв 22 Май 2014 17:41:09  #44 №198475 

>>198474
во второй раз
>будет проверен лишь 1 камень
должно быть
>будут проверены 2 камня

Аноним Чтв 22 Май 2014 17:51:41  #45 №198477 

>>198474
Стратегия для 9: проверяем 2 камня, 2 камня, 2 камня, 1 камень, 1 камень.
Выигрыши: 6, 6, 5, 5, 4, 4, 4, 3, 3. Средний выигрыш 40/9. Как получить 41?

Аноним Чтв 22 Май 2014 17:56:41  #46 №198478 

>>198477
Проверяем 3, дальше 2, дальше 2 и в конце 1.

Аноним Чтв 22 Май 2014 18:14:38  #47 №198480 

>>198448
>Также можно предлагаю обобщить задачу на случай n камней и k радиоактивных (или хотя бы 1000 камней и 2 радиоактивных).
Можно выписать рекуррентные соотношения на ck,ln, где ck,ln, 0<=k<=l<=n - максимальный гарантированный выигрыш для n камней среди которых есть от k до l радиоактивных. Можно ли из этого получить хорошую формулу, как для k=l=1, я не знаю. Но определённо так можно посчитать на компьютере для 1000 камней из которых2 радиоактивные.

Аноним Чтв 22 Май 2014 18:49:16  #48 №198482 

>>198470
Но ведь тогда точка никогда не достигнет другого конца, потому что отрезок расширяется быстрее чем движется точка.

Аноним Чтв 22 Май 2014 18:50:29  #49 №198483 

>>198482
И в момент времени t=inf точка будет в середине отрезка.

Аноним Чтв 22 Май 2014 18:59:10  #50 №198484 

>>198470
Тогда точка достигнет конца отрезка, но медленно и считать ряд в лом :3

Аноним Чтв 22 Май 2014 19:04:06  #51 №198485 

>>198482
я на трупе паскакаля накатал говнопрограмму, всё достигает, так что нехуй. Мне решение увидеть охота.

Аноним Чтв 22 Май 2014 19:05:35  #52 №198486 

>>198485
Скорость точки увеличивается пропорционально удлинению отрезка?

Аноним Чтв 22 Май 2014 19:10:53  #53 №198487 

>>198478
Ага, ты прав, я не прав.

Аноним Чтв 22 Май 2014 19:12:14  #54 №198488 

>>198486
Нет, относительная (в долях отрезка) даже уменьшается, но это не мешает ряду расходится.
мимо-другой

Аноним Чтв 22 Май 2014 19:28:22  #55 №198490 

>>198488
>но это не мешает ряду расходится
И почему тогда
>всё достигает
Кто из вас двоих пиздит?

Аноним Чтв 22 Май 2014 19:42:19  #56 №198492 

>>198490
Э! Ряд расходится, Сумма неограниченно возрастает, точка достигает конца отрезка и уезжает к краю вселенной.
Никто не пиздит.

Аноним Чтв 22 Май 2014 19:54:01  #57 №198494 

>>198492
Как точка может достигнуть конца, если ее скорость в 5 раз меньше скорости расширения отрезка?

Аноним Птн 23 Май 2014 06:32:53  #58 №198546 

>>196225
>насрал под себя

Аноним Птн 23 Май 2014 09:17:08  #59 №198558 

>>198494
>Как точка может достигнуть конца
Мееееедленно

>если ее скорость в 5 раз меньше скорости расширения отрезка
Пруф, что не может, или хуй простой.

Аноним Птн 23 Май 2014 11:26:23  #60 №198579 

>>198318
Почему на оп-пике Путин?

Аноним Птн 23 Май 2014 13:38:21  #61 №198622 

>>198454
>за какое время точка достигнет другого конца отрезка?
Интеграл от f(x)=1/(100+5x) на отрезке [0,1]

sageАноним Птн 23 Май 2014 13:42:44  #62 №198623 

>>198622
Обосрался немного, нужно найти такой отрезок [0,T] на котором упомянутый интеграл равен 1. Т - ответ.

Аноним Птн 23 Май 2014 15:00:10  #63 №198640 
1400842810683.jpg

>>198579
Это не Путин, это Альберт Эйнштейн.

Аноним Птн 23 Май 2014 15:28:56  #64 №198644 

>>198318
Спрошу здесь. По оптимальному управлению с нелинейными минимизируемыми функционалами есть ещё актуальные разработки?

Аноним Птн 23 Май 2014 15:37:17  #65 №198646 

>>198640
Ты охуел что ли это же Гаусс.
http://ru.wikipedia.org/wiki/%D0%9A%D0%B0%D1%80%D0%BB_%D0%93%D0%B0%D1%83%D1%81%D1%81

Аноним Птн 23 Май 2014 15:54:10  #66 №198648 

>>198644
Какие-то новые разработки должны быть - должны же что-то публиковать люди с соответствующих кафедр. Но я подозреваю, что шансы встретить здесь человека с представлением о современном положение дел в области довольно малы, хотя кто знает, может тебе повезёт.

Аноним Птн 23 Май 2014 17:18:13  #67 №198665 

>>198558
>Пруф, что не может, или хуй простой.
Чайник_Рассела.тхт
Сначала ты докажи что может, а потом кукарекай.

Аноним Птн 23 Май 2014 17:20:09  #68 №198666 

>>198646
Этим Гауссом был Альберт Эйнштейн

Аноним Птн 23 Май 2014 18:51:49  #69 №198674 

>>198665
>Не способен в решение
>Не понимает, как ряд с уменьшающимися членами может расходится
>Считает, что кукарека уровня /sci - это не он

Мальчик, что я тебе скажу, что больная (но не смертельно) черепаха, проходящая за первую секунду 1 метр, за вторую - 1/2 метра, за третью - 1/3 метра, за сотую - 1/100 и т.д. способна за достаточно долгое, но конечное время преодолеть расстояние от земли до луны.

Аноним Птн 23 Май 2014 18:54:29  #70 №198675 

>>198622
>>198623
Обосрался ещё чуть-чуть, спутав интеграл непрерывной функции с сумой ряда

Аноним Птн 23 Май 2014 20:06:52  #71 №198682 

>>198674
Интересно, успеет к тепловой смерти Вселенной?

Аноним Птн 23 Май 2014 20:27:17  #72 №198686 

Я мимокрокодил, но я бы разбил на 10 частей и за 10 р проверил бы. 900 гарантированно хороших камней и прибыль в 890 рэ.

Аноним Птн 23 Май 2014 20:54:41  #73 №198694 

>>198674
Ох лол, требовать пруф = неспособен в решение? Ты серьезно?
Ответ "бесконечное время" удовлетворит разве что абстрактных математиков, которые потеряли связь с реальностью. Нормальный человек скажет "никогда".

>>198675
Собственно, предел суммы и есть интеграл.

Аноним Птн 23 Май 2014 21:55:47  #74 №198702 

>>198694
>Собственно, предел суммы и есть интеграл.
Если смотреть на это с общих позиций, то да. Но в данном конкретном случае сумма относится к дискретной формулироке задачи (передвижение и растягивание происходит переменно), а интеграл к непрерывной (передвижение и растягивание происходят равномерно и постоянно).

объясните решение, сап посоны хочу_понять Птн 23 Май 2014 21:56:09  #75 №198703 

Есть семь гирь. Арбуз можно уравновесить тремя гирями, можно четырьмя и можно пяти. Докажите, что одну из гирь набора можно уравновесить несколькими другими

Аноним Суб 24 Май 2014 05:01:10  #76 №198731 

>>198702
Так ведь в задаче-то, насколько я понял, растяжение непрерывное, ведь указана скорость 5 см/секунду которая по умолчанию принимается непрерывной. В задаче не сказано про прерывистое приращение и про очередность приращение/перемещение.

Аноним Суб 24 Май 2014 12:18:24  #77 №198761 

Есть предел, равный нулю. Нужно доказать, что он, собственно, равен нулю. Доказать в разрезе темы числовые ряды. Как такое делать, я уже забыл.

Я могу доказать, что выражение, которое стоит под знаком предела, сходится, значит предел равен нулю, как необходимое условие?

Или как делать.

Предел такой lim [(3n)^n]/[(2n-1)!], n to infty

Аноним Суб 24 Май 2014 13:13:05  #78 №198765 

>>198761
Пиздец ты тупой

Аноним Пнд 26 Май 2014 09:24:21  #79 №199000 

>>198318
Может кто подпробно о Канторовом множестве рассказать или дать ссылку, где понятно написано?

Аноним Пнд 26 Май 2014 12:26:52  #80 №199021 
1401092812440.png

Метематач, Есть одна формула отсюда: ru.wikipedia.org/wiki/%D0%98%D0%BD%D0%B4%D0%B5%D0%BA%D1%81_%D0%BC%D0%B0%D1%81%D1%81%D1%8B_%28%D1%82%D0%B5%D1%85%D0%BD%D0%B8%D1%87%D0%B5%D1%81%D0%BA%D0%B8%D0%B9_%D0%B0%D0%BD%D0%B0%D0%BB%D0%B8%D0%B7%29 .
Вопрос: Почему в этой формуле под суммой нигде не используется переменная i ?
Если бы выражение под суммой совершенно не зависело от i, тогда это было бы эквивалентно простому умножению на n, так же?

Аноним Пнд 26 Май 2014 14:09:06  #81 №199065 

>>198694
>Ответ "бесконечное время"
Бесконечное время - только у тебя, Maria. Читай не жопой:
>достаточно долгое, но конечное время

>>198731
>Так ведь в задаче-то, насколько я понял, растяжение непрерывное
Ну хуй знает:
>Каждую минуту отрезок "растягивается" на 5 см
Я и ещё как минимум 1 анон восприняли это как периодический процесс. В противном случае, кстати, собственное движение точки по ленте становится не очевидно отделить от суммарного.

Аноним Втр 27 Май 2014 00:03:49  #82 №199163 
1401134629709.gif

>>199021
Какой же ты медленный, математач!

Аноним Втр 27 Май 2014 11:57:45  #83 №199212 

>>199021
>Вопрос: Почему в этой формуле под суммой нигде не используется переменная i ?
Потому что она записана маркетингоебом, которые традиционно не могут в математику. EMA и DMA - оконные функции, т.е. cами используют взвешенное суммирование значений в некотором окне вокруг момента t. Сумма тут вообще не нужна. Там же ссылки есть, блджад, перейди и посмотри.

Аноним Срд 28 Май 2014 13:21:39  #84 №199418 

Как найти производную функции от интеграла?

Аноним Срд 28 Май 2014 14:52:48  #85 №199434 
1401274368462.gif

>>199212
Спасибо, хоть один ответил. Но я прошел по всем исходящим ссылкам (их не много) с английской и русской вики (что характерно на других языках этой статьи нет), там нет ничего такого, чего нет в самой вики. Интуитивно прихожу к тому, что i должна задавать смещения аргумента t для EMA и DMA относительно аргумента t самого MassIndex. То есть формула должна выглядеть как на пике (сам рисовал). Я прав?

Аноним Срд 28 Май 2014 15:04:39  #86 №199435 
1401275079995.jpg

>>199418
Интеграл = Первообразная на участке ограниченном пределами интегрирования.
Производная первообразной функции = исходная функция.

Производная интеграла на участке внутри пределов интегрирования = исходная функция.
Производная интеграла на участке за пределами интегрирования не определена.

Аноним Срд 28 Май 2014 17:21:22  #87 №199488 

>>198454
Так мне будет, блять, мой ответ или нет?! С решением, конечно же, няши.

Аноним Срд 28 Май 2014 17:35:37  #88 №199498 

>>199488
Тебе уже ответили: Не достигнет.
>>198485
Ванга: У тебя ошибка в программе.
Или соус в студию.

Аноним Срд 28 Май 2014 17:41:19  #89 №199502 

>>199488
Если у тебя растяжение и движение идет непрерывно или дискретно? Если непрерувно ответ >>198622 с попровкой >>198623 если дискетно то нужно уточнить как именно происходит растяжение и движение, в каком порядке.

Аноним Срд 28 Май 2014 18:14:29  #90 №199516 

>>199498
> соус в студию.
https://ideone.com/rb2u6J

Аноним Срд 28 Май 2014 18:20:44  #91 №199520 

>>198318
Гарантированная прибыль 999. Продаем все камни по одному, по рублю. Тот, кому достанется радиоактивный придет возмущаться и мы вернем ему деньги.
Математики не умеют в деньги.

Аноним Срд 28 Май 2014 18:29:07  #92 №199526 

>>198674
Что если я скажу тебе, что эта черепаха проползет только e метров?

sageАноним Срд 28 Май 2014 18:37:18  #93 №199531 

>>199526
>Что если я скажу тебе, что эта черепаха проползет только e метров?
Я скажу что тогда черепаха сделает еще 1 шаг и проползет е + 1/n метров, где n - номер шага.

Аноним Срд 28 Май 2014 18:46:16  #94 №199537 

>>199531
Наркоман? е метров это при бесконечном времени. Т.е. до е она не доползет.

sageАноним Срд 28 Май 2014 18:50:48  #95 №199538 

>>199537
>Т.е. до е она не доползет
Но ведь ты собрался говорить что она проползет е метров.

Аноним Срд 28 Май 2014 19:05:56  #96 №199545 

>>199538
Больше е метров она не проползет. Тебе такая формулировка нравится?

Аноним Срд 28 Май 2014 19:11:37  #97 №199546 

>>198318
> Как обеспечить максимальную выручку?
Что понимать под выручкой? Просто сумму выплат со стороны заказчика? Или сумма выплат минус плата детектору?
Если первое, то перебирать по одному.

Аноним Срд 28 Май 2014 19:14:20  #98 №199548 

>>199516
Ванга оказалась права, у тебя ошибка в решении:
Попробуй задать входные данные, вместо метров, например в сантиметрах. У тебя измениться результат. С чего бы это? В реальных условиях это тот же физический процесс с теми же величинами.

Аноним Срд 28 Май 2014 19:21:37  #99 №199550 

>>199435
rghost.ru/55924534
второе задание интересует, там должна быть какая-то формула.

Аноним Срд 28 Май 2014 19:27:13  #100 №199552 

>>199548
У меня там в сантиметрах. Вот в миллиметрах https://ideone.com/676OpH - что изменилось? А нихуя.

Аноним Срд 28 Май 2014 21:01:17  #101 №199579 

>>199516
Ошибка в том, что ты рассматриваешь длину отрезка как фиксированную величину в момент передвижения точки:
>position+=speed;
Если точка прыгает не мгновенно, а движется в течении какого-то времени, то за это время, путь, который она должна пройти тоже растягивается вместе с отрезком на котором он отмерен. Этот процесс вообще нельзя симулировать в цикле с дискретными отсчетами.

Аноним Срд 28 Май 2014 21:17:24  #102 №199582 

>>199488
Зачем в этой задаче вообще дана начальная длинна отрезка?
По идее решение должно зависеть только от отношения одной скорости к другой.

Аноним Срд 28 Май 2014 21:25:47  #103 №199584 

>>199579
>Ошибка в том
Прочитай определение слова "ошибка".
>Если точка прыгает не мгновенно
Для непрерывного процесса есть решение. >>198622

Аноним Чтв 29 Май 2014 00:34:37  #104 №199619 
1401309277024.gif

>>198318
Вы там ее решили, нет?
Пусть N = 1000, n камней за раз проверяем. Тогда надо
(N - n) - N/n => max
Производная: N / n^2 - 1
При N=1000, ее корни: +-10*sqrt(10)
Откуда n=32 или 31 - похер,куда округлять, результат один: 1000 - 64 = 936.
Я прав, ОП?

Аноним Чтв 29 Май 2014 09:39:38  #105 №199648 

>>198442
>Таки 1 рубль за проверку 1000 камней одной кучей, и 999 чистого профита после продажи.
таки продать за 1000 не проверяя. они все радиоактивные по условию задачи.

Аноним Чтв 29 Май 2014 10:49:16  #106 №199663 

>>199520
>Тот, кому достанется радиоактивный придет возмущаться
в ОЗПП бьют тревогу, статьи "Анон продал радиоактивные камни!" в газетах, тебя ебут СЭС, МЧС и отдельно ФСБ на предмет "откуда камни взял сука", ты банкротишься, продаёшь почку чтобы расчитаться по внезапно образовавшимся долгам. Аноны не умеет в гешефтмахеры

>>199526
>>199545
Посмотрите на этого унтера, у него ГАРМОНИЧЕСКИЙ РЯД СХОДИТСЯ к е!

>>199619
Твоё решение правильно, но для деление на равные кучки, а оно не оптимально.

Аноним Чтв 29 Май 2014 11:06:43  #107 №199667 
1401347203885.jpg

>>199663
> но для деление

Привет-привет!

Аноним Чтв 29 Май 2014 11:33:47  #108 №199671 

>>199663
>но для деление на равные кучки, а оно не оптимально
У тебя есть доказательство, или только твердая увереность?

Аноним Чтв 29 Май 2014 12:47:19  #109 №199675 

>>199663
> а оно не оптимально.
Таки оптимально.

Ну сначала определимся, что равномерные кучки - это те, где количество отличается максимум на 1.
Пусть N камней разделены k неравномерных кучек, где M - количество в самой большой.
Так как решение гарантированное, то мы проебываем N - M - k.
Перераспределим камни максимально равномерно, так, что кучки будут отличаться максимум на 1 (если N на k нацело не делится). Тогда количество кучек остается k, а максимальное количество M2 может только уменьшиться, то есть M >= M2 (очевидно почему)
Если оно не уменьшилось - то все ок, случай неравномерных кучек перераспределением сводится к равномерному. Если уменьшилось - получаем выигрыш профита на (M - M2) по сравнению с неравномерным.
Откуда вывод - равномерное решение таки оптимально!
>>199619 - в этом решении 32 и 31 дадут очень маленькую кучку в остатке с количеством 8, но перераспределив в нее камней так, чтобы все кучки отличались не более чем на 1, можно заметить, что максимум все равно останется тем-же и количество кучек не изменится.
Более того значения рядом с 31 в некотором диапазоне дают одинаковое значение проеба==64. Можно взять самое честное решение для 29 кучек по 33 и одной по 34, тогда получим 1000 - (30 + 34) все равно равно 936.

>>199619-кун

Аноним Чтв 29 Май 2014 13:08:58  #110 №199684 
1401354538355.png
  1. Так я правильно понял эту формулу >>199434 ?

    2. Как записать пикрелейтед не рекурсивно?
Аноним Чтв 29 Май 2014 16:41:08  #111 №199714 
1401367268445.png

Математик-кун, поясни по хардкору, почему правый базис нельзя перевести непрерывной деформацией в левый? В смысле, интересует, как это можно было бы доказать. Сам физик, вроде интуитивно понятно, но хочется построже. В гугле по верхам ничего по теме не лежит. И будь хорошим, посоветуй, куда читать, чтобы я сам научился доказывать такие вещи, интересно.

Аноним Чтв 29 Май 2014 20:25:07  #112 №199770 

>>199671
>У тебя есть доказательство, или только твердая увереность?
См. выше, там кун решил задачу с помощью компа и паскаля.

Аноним Чтв 29 Май 2014 20:33:33  #113 №199774 

>>199675
В твоём решении есть один изъян: ты почему-то считаешь кучки неупорядоченными и равноценными, в то время как они упорядочены. Контрольный вопрос: как ты распределишь 16 камней и какой получишь гарантированный профит?

Аноним Чтв 29 Май 2014 20:37:17  #114 №199776 

>>199675
Смотри выше, можно достичь 954.

Аноним Чтв 29 Май 2014 22:11:48  #115 №199796 

>>199663
>ГАРМОНИЧЕСКИЙ РЯД СХОДИТСЯ к е
Прошу прощения, я наркоман.

Аноним Птн 30 Май 2014 00:49:17  #116 №199814 

Набыдлокодил стохастическую оптимизацию получилось
33 33 32 32 31 31 30 30 29 29 28 28 27 27 26 26 25 25 24 24 23 23 22 22 21 20 20 20 19 19 18 18 17 17 16 16 15 15 14 14 13 13 12 12 11
Ожидаемая прибыль - 956.672
Очень сильно похоже на оптимум, так как при повторных запусках (каждый запуск это 2900000 рандомных изменений базового) результат не изменяется, иногда попадаются чуть хуже варианты. Вообще похоже что целевая функция непрерывная и с одним экстремумом, тоесть задачу можно решить аналитически нормальными методами оптимизации, но я слишком дно в мат программировании для этого. Если будет не лень, завтра ебну генетическую оптимизацию.
>>199619
Ты не учел что как только мы находим радиоктивный камень - нам не нужно проверять остальные камни. Поэтому выгоднее первые кучки взять большими - так больше шанс что нам попадется радиоактивный камень с самого начала и мы съекономим на проверке всех остальных. Если бы нужно было обязательно проверять каждую кучку (к примеру закон требует что бы каждый продаваемый камень был обязательно проверен продавцом) - то тогда выгоднее делить на максимально равные кучки (как ты и показал), по крайне мере как я это вижу.

Аноним Птн 30 Май 2014 01:12:47  #117 №199815 

>>198674
Ничего удивительно: расстояние от Земли до Луны постоянно (в рамках задачи), в отличии от расстояния между началом отрезка и его концом, которое бесконечно увеличивается.

Аноним Птн 30 Май 2014 01:14:04  #118 №199816 

>>199815
>удивительно
удивительного*
быстрофикс

Аноним Птн 30 Май 2014 09:40:14  #119 №199832 

>>199815
Мах бои! Задача уже решена выше по треду, аналитически. Чего ты теперь хочешь, кроме как кукарекать "НИВИРОЯТНО! ДА БЫТЬ НИ МОЖЕТ"?

Аноним Птн 30 Май 2014 10:03:48  #120 №199833 

>>199832
>НИВИРОЯТНО! ДА БЫТЬ НИ МОЖЕТ
Ну и где я такое говорил, поехавший?

Аноним Птн 30 Май 2014 13:04:04  #121 №199844 
1401440644915.jpg

>>199833
Ясно.

Аноним Птн 30 Май 2014 13:29:54  #122 №199846 

>>198318
1. Сдать 1000 камней на анализ за 1 рубль.
2. Продать 1000 радиоактивных камней в Иран.
3....
4. Profit

Аноним Птн 30 Май 2014 17:04:05  #123 №199869 

>>199844
Ты себя ведешь хуже любого посетителя /по/раши, демагогия на демагогии и демагогией погоняет.

Аноним Птн 30 Май 2014 18:21:48  #124 №199881 

>>199684
Бамп, что ли.

sageАноним Птн 30 Май 2014 20:50:54  #125 №199895 
1401468654657.jpg

У меня вопрос , решал я домашку по алгебре и слаживал матрицы и было задание сложить 2 матрицы: первая- 2*3 , а вторая 3*2 , ну я подумал и вспомнил что можно нулевой столбец или строку добавить, так и сделал. Так вот мне препод зачеркнул мой вариант и сказал что просто нельзя. Получил 4 , так все-таки почему нельзя то?да, я экономист

sageАноним Птн 30 Май 2014 21:18:41  #126 №199897 

>>199895
У меня вопрос, сводил я баланс и нужно было свести 230000 дебита с 2300000 кредита, ну я подумал и вспомнил что можно нолик дописать. Так вот инспектор зачеркнул мой баланс и сказал что просто нельзя. Получил 4 года, так все-таки почему нельзя-то?

Аноним Птн 30 Май 2014 21:41:24  #127 №199902 

>>199897
Задача была не экономическая, да и прибавление нулевой строки в таблице ничего не изменит

Аноним Птн 30 Май 2014 21:43:48  #128 №199903 
1401471828404.jpg

Я так понимаю, я попал по адресу. В общем суть в том, что нужны ответы на некоторые вопросы по статистике, а сам я её уже ничерта не помню. Сопсна, вопросы:
1. Условия построения дискретного вариационного ряда. (обьём выборки, удалённость значений друг от друга).
2. Условия выбора критерия Лапласа для проверки гипотезы о равенстве двух средних.
3. И уж совсем пиздец - пикрилейтед.
Многоуважаемый анон, как ты мог догадаться, я решаю тесты по математике, которые сделала криворукая блядина, наивно считающая себя не только преподавателем математики, но и информатики(!)

Аноним Суб 31 Май 2014 01:43:12  #129 №199917 

>>199903
А ну-ка на хуй пошёл, мразь с домашкой.

Аноним Суб 31 Май 2014 02:42:38  #130 №199919 
1401489758547.jpg

>>199895
>решал я домашку по алгебре и слаживал
>решал и слаживал

Аноним Суб 31 Май 2014 02:54:31  #131 №199920 
1401490471775.jpg

А реклама-то в /бэ работает.
^_^

Аноним Суб 31 Май 2014 06:17:10  #132 №199924 

>>199917

А ну-ка решил мне БЫСТРАБЛЯДЬ эту хуйню. Я сюда за советом пришёл, а не разводить интернет-баталии с какимто аноном, который нихуя не сможет посоветовать, но сможет нахуй послать, потому что он тупо не знает. Домашка не моя, сам бы решил, но тупо не помню статистическую математику, ибо уже и не студент нихера, давно это было. Ты бы сначала посоветовал, а потом бы уже и нахуй посылал. Тред то про математику, или где?

sageАноним Суб 31 Май 2014 10:17:13  #133 №199928 

>>199924
Ты сюда пришел за болью и унижением.
Иди нахуй.

sageАноним Суб 31 Май 2014 10:51:45  #134 №199929 

>>199924
>статистическую математику
Проиграл с этой корзиночки

Аноним Суб 31 Май 2014 11:57:39  #135 №199932 
1401523059384.jpg

>>199924
>аноном

Аноним Суб 31 Май 2014 15:46:02  #136 №199953 

>>199924
> тупо не помню
> , ибо уже и не студент нихера, давно это было
Вся суть типичного хомячка-образованца. Когда дорастаешь до использования знаний и они кроме того оказываются нужными при возникновении посторонних вопрос/проблем/задач, наш обучившийся учиться уже нихуя и не помнит, а прочитать заново нам лень. Поссал хорошенько на ментально немощное быдло. Кто не использует хотя бы метод расширения интервалов для надежного занесения вещей в долговременную память - не человек. Гоните его, насмехайтесь.

Аноним Суб 31 Май 2014 16:50:58  #137 №199958 

Привет, анон. Скажи, а какую функцию в ZFC выполняют аксиомы сепарации и объединения? Ведь первая является следствием существования нуль-сета, а вторая самоочевидна из фундаментального отношения a ∈ А.

sageАноним Суб 31 Май 2014 17:11:03  #138 №199961 

>>199924
Ты как со мной разговариваешь, пидор? Просишь решить твое школьное говно и еще выебываешься? Перед мамкой своим короткостволом выёбываться будешь, когда она тебе расскажет какой маленький хуй твоего директора отсосала за твой аттестат без двоек.

Аноним Суб 31 Май 2014 17:14:17  #139 №199962 

>>199961

Ну это же анонимная борда, здесь вся суть в том, что можно выражаться как хочешь, не стесняясь, а люди не будут от этого баттхёртить, а будут реагировать адекватно. Да и такие как ты всёравно в итоге не помогают, только важничают.

Аноним Суб 31 Май 2014 17:36:09  #140 №199963 
1401543369694.jpg

>>199962
>анонимная борда
>вся суть

Аноним Суб 31 Май 2014 18:02:13  #141 №199964 
1401544933142.jpg

>>199962
Слыш, уёбок, пасть заткни уже. Ты не охуел столько пиздеть? Тебя по хуям потаскали? Что ты ещё ждёшь, выхухоль?

Аноним Суб 31 Май 2014 18:23:40  #142 №199966 

Аноняши, посоветуйте годных лекций по матану или не только (дискретка и прочее тоже будет в тему, очевидно), чтобы прокачаться годно. У меня второй курс технического вуза, но там хуйню какую-то дают, хотет годноты.

Аноним Суб 31 Май 2014 20:06:36  #143 №199974 
1401552396503.jpg

>>199962
Вся суть - школьник думает, что анонимная борда для того, чтобы других посылать нахуй. Пиздуй в свой МДК, днина ебаная)))

Аноним Суб 31 Май 2014 20:24:54  #144 №199979 
1401553494667.jpg

>>199974
>анонимная борда

Аноним Суб 31 Май 2014 22:16:54  #145 №199986 

Привет, анон. Поясни по хардкору за производную и интеграл. Нахуя они нужны? Для дебила, позязя.

Аноним Суб 31 Май 2014 22:40:15  #146 №199988 
1401561615318.jpg

>>199986

Аноним Суб 31 Май 2014 22:44:14  #147 №199989 

>>199988
Ну вкратце хотя бы.

Аноним Суб 31 Май 2014 23:09:17  #148 №199990 

>>199986
>Нахуя они нужны?
Ну хуй знает. Люди на своем веку хуйней маялись и выдумали такую штуку как функцию - зависимость одних величин от других. Так вот, производная это такая херня (порой тоже сама по себе функция) по отношению к функции, которая показывает скорость изменения одной величины от другой, а интегралл это пиздец что, но по сути сумма произведений бесконечно малых величин, проще его представить как площадь ограниченную графиком функции...

Эти фигни порой полезны при расчетах всякой хитроменяющейся поеботни. Кроче открой блядь учебник и прочитай, это самая легкая херня что есть в алгебре, легче уж только пределы.

Аноним Суб 31 Май 2014 23:11:34  #149 №199991 

>>199990
А интеграл по поверхности нахуй нужен? Это площадь поверхности или что?

Аноним Вск 01 Июн 2014 02:02:18  #150 №200007 

>>199869
я смотрю ты хорошо знаком с /po/, почему бы тебе не съебать туда?

Аноним Вск 01 Июн 2014 13:24:35  #151 №200015 

>>199990
>в алгебре
В матане, при чём тут алгебра?

Аноним Вск 01 Июн 2014 15:44:39  #152 №200023 

Объясните, почему предел n-ной степени из n (а так же из а) при n->inf равен 1? В учебнике это показано из "доказанного", но я не нашел связи с предыдущими выкладками. Во втором случае интуитивно понятно, что (1+eps)^n рано или поздно достигнет фиксированного а, но в первом подкоренное значение само стремится к бесконечности.

Аноним Вск 01 Июн 2014 16:20:54  #153 №200025 

>>200023
n представляется в виде (1 + (n^(1/n) -1) )^n это раскладывается в бином ньютона и получается ( n(n-1)/2 )*(n^(1/n) - 1)^2 - эта хуйня всегда меньше n, тоест ьполучается неравенство
n > ( n(n-1)/2 )*(n^(1/n) - 1)^2 откуда
(n^(1/n) - 1)^2 < 2/(n-1)
|n^(1/n) - 1| < (2/(n-1))^2 предел (2/(n-1))^2 равен 0, следовательно предел |n^(1/n) - 1| также следовательно предел n^(1/n) еденица.
Поищи в интеренете ормальное доказательство, на двачике формулы неудобно писать.

Аноним Вск 01 Июн 2014 20:32:19  #154 №200035 

>>199958
Очевидно же, эти аксиомы служат для построения новых видов множеств.
>Ведь первая является следствием существования нуль-сета, а вторая самоочевидна из фундаментального отношения a ∈ А.
Ловите поехавшего.

Аноним Пнд 02 Июн 2014 11:55:34  #155 №200103 

>>200035

Но что мешает строить новые множества просто так, без этих аксиом? Ведь они не необходимы, чтобы знать, что такая возможность есть. И так ничего не мешает сделать из сета и свойства новый сет, соответствующая аксиома ничего нового не привносит.

Аноним Пнд 02 Июн 2014 14:16:02  #156 №200120 

>>200103
>Но что мешает строить новые множества просто так, без этих аксиом?
Ну, как бы сказать, мешает отсутствие правила согласно которому образуются новые множества. А правила эти задаются аксиомами. Ты не ретард часом?

Аноним Пнд 02 Июн 2014 19:32:23  #157 №200161 
1401723143528.jpg

Как-бы задам здесь свой вопрос.Имеет некое отношение к матану, и поэтому не пошел в тред тупых вопросов.
На пике, график силы тока в цепи, в которой два кенотрона, выпрямительные лампы.Сила тока, есть естественная величина, а значит дифференцируема.
Но на графике, на самых нижних -острых- точках, функция не дифференцируема, касательной в этих точках не существует, значит нет и производной.Но величина-то естественная, а значит и дифференцируема.
Чому так?Может ли естественная величина так резко изменяться?
Кто может объяснить?

Аноним Пнд 02 Июн 2014 21:14:46  #158 №200174 
1401729286826.jpg

>>200161
У тебя пик неправильный.

Аноним Пнд 02 Июн 2014 21:27:36  #159 №200177 

>>200120

Но ведь это просто условщина и формальность. А как мы этот изначальный сет А получаем тогда?

Аноним Пнд 02 Июн 2014 22:02:58  #160 №200182 

>>200177
>аксиома пустого множества

Аноним Пнд 02 Июн 2014 22:41:09  #161 №200184 

>>200174
Двачую, в книжке график дан примерный, мол, изменения между этими волнами пренебрежимо малы. А на деле там всё-таки не так резко меняется, а постепенно, хоть и очень быстро.

Аноним Втр 03 Июн 2014 04:10:08  #162 №200218 

>>200161
>Но величина-то естественная, а значит и дифференцируема.
По идее, там за счет емкости и индуктивности элементов будет гладкий график. В расчетах-то они за нуль берутся, небось.

Аноним Втр 03 Июн 2014 04:17:22  #163 №200220 

>>200218
Тьфу, хуйню сморозил. Имел в виду, что реактивное сопротивление за нуль там скорее всего берется.

Аноним Втр 03 Июн 2014 05:02:48  #164 №200228 

>>200220
А, ну и еще ВАХ всякихъ йоба-ламп тоже в реальной жизни не идеальная.

Аноним Втр 03 Июн 2014 10:09:08  #165 №200235 

>>200161
На пике отскок упругого шарик, с точностью до осей он же - график X(t). Пространственное положение шарика, есть естественная величина, а значит дифференцируема. Но на графике, на самой нижней -острой- точке, функция не дифференцируема, касательной в этих точках не существует, значит нет и производной.Но величина-то естественная, а значит и дифференцируема.
Чому так?Может ли естественная величина так резко изменяться?
Кто может объяснить?

Аноним Втр 03 Июн 2014 10:12:43  #166 №200236 
1401775963955.png

>>200235
пик отклеился

Аноним Втр 03 Июн 2014 10:18:20  #167 №200237 

>>200235
А можно я, можно я!

Разгадка в том, что в реальности идеально твердых штук не бывает, при отскоке шарик сплющится — и его центр при этом опишет не угол, а что-то такое плавное.

sageАноним Втр 03 Июн 2014 11:05:46  #168 №200240 

>>200235
total eblan
Элементарные функции могут быть непрерывны в некоторой точке, но не быть в ней дифференцируемы. Напр., функция f(x) = |x| является непрерывной на всей вещественной оси, но её производная испытывает скачок при переходе через точку x=0, в котором эта функция не является дифференцируемой. В этой точке нельзя провести и касательную к графику функции.

Аноним Втр 03 Июн 2014 11:58:28  #169 №200242 

>>200237
>Разгадка в том, что в реальности идеально твердых штук не бывает
>в реальности идеальных штук вообще не бывает
THIS!

>>200240
Если ты осилишь посмотреть тред, то тут речь идёт о тезисе "функции естественных физических величин дифференцируемы по времени", а не "произвольные непрерывные функции дифференцируемы". В остальном - ты прав, спасибо Капитан Матанализ.

Аноним Втр 03 Июн 2014 12:07:28  #170 №200243 

>>200242
>речь идёт о тезисе "функции естественных физических величин дифференцируемы по времени"
хуевый тезис

Аноним Втр 03 Июн 2014 13:55:55  #171 №200251 
1401789355645.png

>>200174
Пик правильный, но в цепи два кенотрона, и ток непрерывный.При этом разница фаз меж ними равна 180 градусам.
Но вопрос не в этом.Меня более интересует тезис "функции естественных физических величин дифференцируемы по времени"
Действителен ли он?Чому так?

Аноним Втр 03 Июн 2014 14:47:20  #172 №200254 

>>198318
Объясните, как ряд с общим членом, стремящимся к нулю может расходиться?

Аноним Втр 03 Июн 2014 15:38:46  #173 №200263 

>>200243
Я бы так не сказал, клоун.

Аноним Втр 03 Июн 2014 15:50:54  #174 №200265 

>>200254
Ну вот так.
1/2 + 1/3 + 1/4 ...

Аноним Втр 03 Июн 2014 16:19:37  #175 №200274 

>>200263
И с хуя бы? Прикинь, функция вообще нихуя не обязана быть непрерывной - цепь разомкнута, тока нет, цепь замкнули, ток пошел, на краю графика функция дифференцируется? Точно так же и с прохождением через ноль. Но это в идеальных условиях, в реальности нихуя не будет работать - паразитные емкости, индуктивности, долбоебы вроде тебя, которые все испортят. Вывод хуевый тезис, положи где взял

Аноним Втр 03 Июн 2014 17:55:21  #176 №200283 

>>200274
>цепь разомкнута, тока нет, цепь замкнули, ток пошел, на краю графика функция дифференцируется?
Каком краю графика, бака? В твоем мысленном эксперименте ток равен нулю на (-inf; 0], а затем начинает плавно возрастать.

Аноним Втр 03 Июн 2014 19:00:00  #177 №200288 

>>200283
>плавно
лоллировал с этого физика

Аноним Втр 03 Июн 2014 20:11:17  #178 №200298 

>>200288
Не слышал про гладкие неаналитические функции, клоун?

Аноним Втр 03 Июн 2014 20:59:22  #179 №200305 

>>200298
Но ведь так называемая функция физической величины от времени, в силу невозможности совершенно точно измерить физическую величину, всегда является аппроксимацией, претендующей на истинность с какой-то вероятностью.
О дифференцируемости этой "истинной" функции мы ничего сказать не можем, а аппроксимация может быть либо дифференцируема в каких-то точках, либо нет в зависимости от выбранной модели.

Аноним Срд 04 Июн 2014 00:42:19  #180 №200332 

>>200305
Внезапно истинные функции могут оказаться вообще дискретными

Аноним Срд 04 Июн 2014 00:43:11  #181 №200333 
1401828191844.png

Очевидно я недопонимаю сути рядов. Корень n-ной степени из а>0 при n->inf равен 1. Почему эта подпоследовательность может быть и >1 и <1?

Аноним Срд 04 Июн 2014 00:45:58  #182 №200334 

>>200333
И вообще, здесь несколько тредов о математике, в каком лучше задавать вопросы для получения ответов по делу?

Аноним Срд 04 Июн 2014 00:57:08  #183 №200336 

>>200333
Там не a, а a_n, которые разные для разных n.

Аноним Срд 04 Июн 2014 08:18:30  #184 №200366 

>>200182

Во-первых, такой нет, во-вторых, и что ты из неё сепарацией получишь?

Аноним Срд 04 Июн 2014 18:27:23  #185 №200477 

Есть ИТТ кто-нибудь, кто занимается по этой программе? http://imperium.lenin.ru/~verbit/MATH/programma.html

Аноним Срд 04 Июн 2014 21:38:31  #186 №200535 
1401903511069.png

Сумма каких отрезков? Я понимаю, что p = |ML| + |PB|, но почему тогда |ML| = |AB|?

Аноним Чтв 05 Июн 2014 09:57:04  #187 №200611 

>>200535
Что-то я ночью тупил, сейчас сразу всё понял (равенство треугольников). Вопрос снят.

ОП Чтв 05 Июн 2014 16:49:11  #188 №200660 

>>199895
У меня есть для тебя два вопроса, надеюсь, что ответив на них и прочитав свой ответ, ты поймёшь, почему можно или почему нельзя.

1. Что изменит добавление ноулевого столбика в матрицу 2*3 или 3*2?
2.Какая была задача?

Аноним Чтв 05 Июн 2014 16:52:57  #189 №200661 

Шучу, не ОП.

ОПчик, я извиняюся, графу имя не подтёр.

Аноним Чтв 05 Июн 2014 16:58:28  #190 №200663 
1401973108625.png

В предпоследней строчке имеем a^x = a^(x0)(a^(x-x0)-1), но если раскрыть скобки, то получается a^x = a^x - a^(x0). Алсо, т.к. |a^(x-x0) - 1| < εa^(-x0), тогда a^(x0)(a^(x-x0)-1) < a^(x0)εa^(-x0), т.е. получается, что a^x < ε. Я снова туплю или тут ошибка?

Аноним Чтв 05 Июн 2014 17:00:52  #191 №200664 

>>200015
>>199991
>>199990
Я недоучка переумник, не серчайте, если что. Только вот современная, самая распространённая на шарике десятичная система счисления начтолько двоичноубога и крупномасштабна, что при её использовании приближённость всех без исключения значений не даёт продвигаться гармонично в развитии науки всей и полностью?
Экономисты, наверное, это видят очень хорошо, изучая бухучёты и прочие заковыки современной экономики.
Не пора ли уже переходить массово на более точные=продуктивные системы счисления, как Вы думаете, товарищи математАны?

sageАноним Чтв 05 Июн 2014 17:04:34  #192 №200665 

>>200664
>переходить массово на более точные=продуктивные системы счисления
Это те что с основнием е?

Аноним Чтв 05 Июн 2014 17:09:10  #193 №200667 
1401973750178.png

И в догонку. В частном сменился знак с + на -, разве это не усилило неравенство с <= на <?

Аноним Чтв 05 Июн 2014 17:57:11  #194 №200670 

Какой смысл имеет последователь элемента, который не является множеством? Чему он будет равен?

Аноним Чтв 05 Июн 2014 18:24:32  #195 №200673 

>>200665
Да, скорее всего или те, что способны наиболее полно
описать сферические или икосаэдральновидные (те, что образуют пузыри при взаимном расположении в пространстве, не оставляющем незанятого ими места(2 пузыря, к примеру, образуют 2 неполные сферы с 1 касательной к ним плоскостью)). Ну, то есть я исхожу из основ нашей любой, и эфирной (точнее информационной) деятельности - биологических закономерностях построения всяких структур.
К слову о структурах, современное состояние архитектуры гоаорит или о грубейшем нарушении принципов строения живой материи, или о глобальном самоопроеделении всяких структур человеческой деятельности в общем механизме жизни биосферы. (стремление к округлости присутствует на всех уровнях живой материи, в той или иной степени, а кристаллические структуры - суть остановка развития эффективного).
Надеюсь рамки математического лексикона я соблюл.

Аноним Чтв 05 Июн 2014 18:28:18  #196 №200674 

>>200667
Откуда отрывок?
мимосамообразованец

Аноним Чтв 05 Июн 2014 19:10:13  #197 №200680 

>>200674
Математический анализ (Зорич), часть 1.

Аноним Чтв 05 Июн 2014 19:12:04  #198 №200683 

>>200674
Очевидный Зорич очевиден.

Аноним Птн 06 Июн 2014 10:50:47  #199 №200762 
1402037447020.png

Извините что принес сюда эту хуйню уровня 5 класса.
Но поясните за задачу, может я не понял условие? Я понял так: в 24 Тиффани будет в 3 раза старше чем она сейчас, тогда очевидно ей 8 лет. Но ответ 12! Как правильно звучит условие, я перезанимался уже сегодня, голова кипит.

Аноним Птн 06 Июн 2014 10:53:10  #200 №200763 

>>200762
Открыл картинку и понял какой я даун блять. Пиздец так обосраться. Удаляюсь.

Аноним Птн 06 Июн 2014 11:22:37  #201 №200769 

>>200762
"In 24 years" значит "Через 24 года".

Аноним Птн 06 Июн 2014 14:04:01  #202 №200801 

>>200663
>>200667
Мне больше не хотят помогать?

Аноним Птн 06 Июн 2014 15:39:53  #203 №200826 

>>200801
А почему тут >>200667 должно было усилить? Первый модуль преобразуется согласно св-вам модуля (нестрогое неравенство), второй модуль преобразуется тождественно согласно выкладкам выше.

Аноним Птн 06 Июн 2014 15:47:29  #204 №200827 

Аноны, куда лучше направить трактор среди стран европы, дабы заниматься спокойно математикой и/или теоретической физикой? Если ли тут такие поехавшие уехавшие?

Аноним Птн 06 Июн 2014 16:29:14  #205 №200832 

>>200827

Дефайн спокойно.

Аноним Птн 06 Июн 2014 21:11:45  #206 №200881 

>>198318
квиксорт
мимохуй

Аноним Птн 06 Июн 2014 22:32:00  #207 №200902 

>>198351
>Тогда ты с вероятность 999/1000 потратишь 999 рублей на проверку
Нихуя, вероятностное распределение нужно смотреть.
Есть шанс, что и первый взятый камень окажется радиоактивным, или второй. Или сотый. Или семисотый. В любом из перечисленных случаев ты что-то получишь, только в случае с тысячным радиоактивным камнем убыток будет 1 рубль.

Аноним Суб 07 Июн 2014 01:34:47  #208 №200941 

>>198318
Кольца вычетов - конечные циклические группы?

Аноним Суб 07 Июн 2014 01:52:43  #209 №200945 

Анон, помоги. У лингвист-куна завтра зачет по мат. обработке (не знаю, нахуй она ему нужна). Осталась одна лаба. Он-то гуманитарий ссаный и нихуя не понимает в этом, а вам техно-богам это как два пальца обмочить. В общем, задание такое: есть два стула две группы. Одна сдала тест на такие оценки: 2,2,3,3,3,4,5, а другая на такие: 4,4,4,4,4,5,5. Нужно посчитать коэффициент корреляции в "exel", но это - хуйня. С этим я справился - не совсем долбоёб еще. Коэффициент=0,867227 НО, блядь, ей надо еще вывод сделать. Тут я попал... Я же нихуя не понимаю, что такое, блядь, корреляция(гуглил, читал - не помогло). Так вот, если кому не лень, напишите мне этот блядский вывод, ребят. Достаточно 5 предложений. Я в вас верю. Аллаху Акбар.

Аноним Суб 07 Июн 2014 02:08:28  #210 №200949 

>>200826
Потому что из-за минуса знаменатель дроби справа стал меньше, а дробь больше.

Аноним Суб 07 Июн 2014 02:38:07  #211 №200952 
1402094287614.gif

Анон, можешь ли ты объяснить мне суть логарифмов на интуитивном уровне? Я очень хотел бы увидеть их так же ясно, как вижу. к примеру, теорему Пифагора с разрезанным на треугольники квадратом и не только понять их, но и прочувствовать. Я все пытаюсь представить себе сам процесс, но ничего не получается, наверное я тупой :с

Аноним Суб 07 Июн 2014 03:00:54  #212 №200954 

>>200952
Аноны, раз такой вопрос поднялся, посоветуйте годную книгу по геометрии. Желательно с направлением на применение irl.

Аноним Суб 07 Июн 2014 03:52:54  #213 №200958 

>>200949
>Потому что из-за минуса знаменатель дроби, возможно, справа стал меньше, а дробь, возможно, больше.

Аноним Суб 07 Июн 2014 07:40:41  #214 №200970 

>>200952
Если нужно геометрическое представление, то ln(a) - это площадь под кривой 1/х, от х равного 1 до х равного а. Нетрудно догадаться, что площадь под 1/х от а до аb равна площади от 1 до b - надо только сжать ось х в а раз, а ось y растянуть в те же а раз - площадь не изменится. Из этого следует что ln(ab)равно ln(a)+ln(b).

Аноним Суб 07 Июн 2014 08:59:38  #215 №200973 

>>200952
Логарифм — это в какую степень нужно возвести основние, чтобы получить данное число. Логарифм 8 по основанию2 — 3

Аноним Суб 07 Июн 2014 11:03:09  #216 №200974 

>>200945
Задание гавно. Кореляцию чего ты с чем считал? Хуита полная.

Аноним Суб 07 Июн 2014 11:21:54  #217 №200975 

>>200954
>с направлением на применение irl
Как ты себе это представляешь? Задачки типа "Посчитать расстояние от дома до магазина по теореме косинусов"? И ещё зависит от того, что больше интересует -- теоретический курс геометрии или задачник. Если последнее, то недавно вышел неплохой сборник Шеня http://www.mccme.ru/free-books/#shen-geo

Аноним Суб 07 Июн 2014 13:00:14  #218 №200980 

>>200974
Думаешь, я знаю? Посчитал по формуле и все. Там короче, что-то надо было написать типо "если корреляция больше того-то, то будет то-то, если меньше, то - то-то". Мне уже одногруппница все написала. Спасибо.

Аноним Суб 07 Июн 2014 16:11:05  #219 №200996 

>>200975
> Как ты себе это представляешь?
Ну так и представляю. В детстве у меня была советская книга, где объяснялось как рассчитать расстояния до какого-то объекта на море, узнать высоту дерева, используя только кусок говна и прочие задания. Ну и все это с конкретными теоремами вперемешку. Только вот автора и название не помню. Думаю, должно же быть много подобного.

Аноним Суб 07 Июн 2014 16:30:03  #220 №200998 

Аноны-математики, как вы учите английский?
Хотелось бы улучшить знание языка, чтобы читать статьи на английском (по математики и вообще).
Задаю вопрос здесь, т.к. рекомендации вроде лингвалео от господ гуманитариев не для меня.

Аноним Суб 07 Июн 2014 17:28:22  #221 №201006 

>>198318
Русский бизнесмен в треде.
Максимальная выручка колеблется между 999 и 1000 рублями. Продаете все камни, повышенных обязательств на себя не берете мол, найдете косяк - поменяем, лол, если лох обнаружит радиоактивный и захочет ебаться с возвратом за 1 рубль, немного мурыжите его там, заява, экспертиза и тп и возвращаете деньги.

Аноним Суб 07 Июн 2014 21:01:55  #222 №201033 

>>200998
Лингвалео - хуйня для высасывания денег. Толком там ничему научишься. Ну во первых, чтобы просто читать статьи на английском, нужно обладать минимумом знания грамматики (9 основных времен, даже меньше), который осваивается за неделю. Употреблять их на письме и в разговорной речи ты, конечно, все равно не сможешь, но понимать их ты будешь. Ну и вокабуляр. Ищешь где нибудь статью с толкованием большинства математических терминов на английском и учишь. Но это, конечно, если ты уже хоть немного можешь в английском. Если нет, то читай лучше сначала просто любую адаптированную литературу(советую сказки, они интересные). Начинай с "pre intermediate", выписывай слова, которые не знвешь. Попутно изучай вокабуляр по спец. книгам, там обычно даны распространенные темы, по ним слова, устойчивые выражения, фразовые глаголы. Эти книги тоже распределены по уровням, бери сразу "intertmediate". Книг таких достаточно много. Обычно называются "Vocabulary и что-то там еще". Желательно, чтобы толкования слов там были тоже на английском, но если совсем не могешь, то можно и на русском, но я таких книг не встречал, возможно их даже нет. Когда поймешь, что "pre intermediate" тебе дается очень легко, переходи сразу на "upper intrmediate" ("intrmediate" можешь пропустить) и делай тоже самое. Затем "advanced", а затем можешь начинать читать короткие истории в оригинале. Советую опять-же сказки. Я в универе читал Сарояна, он много писал про несправедливость и про одиночество, мне нравилось. Они короткие и заставляют подумать. Ну и через год можешь начинать читать свои статьи. Опять же ты сможешь только читать. Говорить и писать ты не сможешь нихуя.
Лингвист-кун

Аноним Суб 07 Июн 2014 21:09:09  #223 №201034 

>>201033
Еще один момент. На грамматику можно забить, но не не совсем. Времена и модальные и основные варианты их использования выучить придется. Для этого советую нанять учителя.

Аноним Суб 07 Июн 2014 21:10:11  #224 №201035 

>>201034
Лингвист-кун

Аноним Суб 07 Июн 2014 21:43:51  #225 №201037 

>>201034

> нанять учителя

Таких барыг сразу нахуй посылайте, анончики. Это всё быдло, стремящееся срубить бабла на незнающих людях. Прекрасно любой язык учиться в одиночку.

Аноним Суб 07 Июн 2014 21:44:51  #226 №201038 

>>201037

> учится

Пофиксил, чтоб не было "ололо я вижу как ты русский в одиночку выучил".

Аноним Суб 07 Июн 2014 21:54:24  #227 №201040 

>>200670

Никто так и не ответил. Всё ясно с вами. Вот еще два вопроса:

Всякий ли сет кроме нуль-сета содержит нуль-сет?

Могут ли неупорядоченные множества содержать несколько одинаковых элементов? Ведь в противном случае и упорядоченные множества не смогут.

Аноним Суб 07 Июн 2014 22:01:11  #228 №201042 

>>200998
У меня было примерно так. В школе и на первых курсах университета я английский толком не выучил (типичная оценка 3-4). Когда я начал читать математические тексты на английском, я не пытался понять точный смысл каждого отдельного предложения, а разбирался в математическом существе текста, исходя из того, что я всё-таки понимал. После того, как я разобрался в нескольких статьях и одном учебнике (страниц 500 в сумме) проблемы с пониманием математических текстов исчезли.

Исходя из своего опыта могу посоветовать начать с текстов, которые с одной стороны интересны тебе с математической точки зрения и с другой содержание которых ожидается быть достаточно понятным.

Аноним Суб 07 Июн 2014 22:08:21  #229 №201043 

Я уверен, что ты не знаешь ничего кроме "London is the capital of Great Britain." Да, и как ты собрался учиться разговаривать в одиночку? Кто будет проверять правильность твоей артикуляции? Интонации? Как человек свободно владеющий английским и понимающий испанский заявляю - это нереально сделать самому. Я не советовал учить все с преподом, я посоветовал выучить с ним только основы грамматики. А вокабуляр же, что является самым сложнвм вполне можно учить самому. Грамматику тоже можно самому, но когда ты не знаешь нихуя это очень сложно. С хорошим учителем это будет в 5 раз легче. Так что, обоссы себе ебало.
Лингвист-кун

Аноним Суб 07 Июн 2014 22:17:15  #230 №201044 

>>200958
> знаменатель дроби, возможно, справа стал меньше
Почему возможно? У нас 0< β/ỹ < 1, т.е. после раскрытия модуля знак останется положительным. Пусть, для наглядности, β/ỹ = 0,1, тогда слева имеем 1/(1+0,1) = 10/11, а справа 1/(1-0,1) = 10/9. Справа эта дробь больше единицы, а слева меньше.
Алсо первый вопрос меня не меньше интересует, у меня там вообще ерунда выходит.

Аноним Суб 07 Июн 2014 22:22:50  #231 №201045 

>>201042
Да хуй знает. Мне кажется, если мне дать на перевод текст с обилием матемптических терминов, я нихуя не переведу по-человечески. Может, потому что нихуя не понимаю в математике.
Лингвист-кун

Аноним Суб 07 Июн 2014 22:30:11  #232 №201046 

>>201040
Всякое чисто множество состоит из особым образом скомпонованных пустых множеств.

Аноним Суб 07 Июн 2014 22:58:15  #233 №201052 

>>201037
>Это всё быдло
быдло это ты
>Прекрасно любой язык учиться в одиночку
сначала выучи хотя бы один восточный язык в одиночку, а потом уже скажешь это
Язык можно учить в одиночку, когда ты уже его хорошо знаешь, то есть его можно только совершенствовать

Аноним Суб 07 Июн 2014 23:07:23  #234 №201055 
[url]

Да вся ваша математика-хуятика всё хуйня для задротов

sageАноним Суб 07 Июн 2014 23:07:34  #235 №201056 
[url]

Давайте что ли отвлечемся немного, посмеёмся над ущербным.

Аноним Суб 07 Июн 2014 23:07:45  #236 №201057 

>>201046

> чисто множество

Что такое?

Аноним Суб 07 Июн 2014 23:11:06  #237 №201059 

>>201043

Про аудиоприложения к книжкам и диктофон не слышал? Конечно, долбоёбу с гуманитарием головного мозга и такое нереальным окажется.
Иди пиарся в другом месте, в этом тебе не рады.

>>201052

Ну хорошо допускаю, что с восточными может быть не так. Но и тут сомневаюсь.
А теперь съеби, мелкобуквенный.

Аноним Суб 07 Июн 2014 23:28:51  #238 №201061 

>>201059
Ну и за какое время ты выучил английский в одиночку, чтобы свободно читать без словаря художественную или техническую литературу? Можно также учить самостоятельно второй язык, но похожий на предыдущий. Я так учил испанский, но говорю один хуй плохо.
По поводу восточных языков. Я знаю людей, которые прекрасно знают английский или немецкий, но выбрав вторым языком китайский/японский, пожалели, ибо это жопа. За 4 года изучения их в вузе с носителем, они научились только заказывать кофе в кафе. А ты говоришь самостоятельно...

Аноним Суб 07 Июн 2014 23:32:21  #239 №201063 

>>201059
Знаю, конечно, и тех, кто хорошо знает китайский, но эти люди ездили по несколько на полугодичные стажировки в Китай.

Аноним Вск 08 Июн 2014 00:05:06  #240 №201070 

>>201061

Лет 5 примерно, но не стоит забывать, что я при этом совершенно целью выучить язык не задавался. То есть я ходил себе в школу, смотрел кинцо и рубал видю, читал статейки время от времени, он и выучился. Только не говори теперь "ололо ну ты же с учителем йоба", да толку от того школьного обучения - ноль целых хуй десятых был. Объясняли элементарщину миллион раз специально для даунов. Никакого профита от того что "жывой учительже" не было.

Значит дауны, очевидно же. Охуеть, уже им и носитель, и четыре года, а они все равно ниасилили. Тут уж ничего не поделать, если человек - дэбил, то это надолго.

>>201063

И правильно делали. Вот это грамотная инвестиция, а не брать на ротан у личных тренеров, даже если это репетиторы. Это для сакеров, меток.

Аноним Вск 08 Июн 2014 00:38:40  #241 №201073 

>>201070
>То есть я ходил себе в школу, смотрел кинцо и рубал видю, читал статейки время от времени, он и выучился.
Ок, я поверю, что за 5 лет ты научился понимать язык таким методом. Но признайся, ты ведь не можешь грамотно поддержать диалог или высказать грамотно мнение по поводу чего-либо? Почему ты так отрицаешь преподавателя? Это же реально проще начинать с учителем. Он видит твои слабые стороны и объяснит то, что ты не понимаешь. А по поводу тех, кто за 4 года нихера не выучил, я сказал к тому, что его нереально выучить на хорошем уровне без стажировки. А ты мне тут про самостоятельное изучение загинаешь.
>Это всё быдло, стремящееся срубить бабла на незнающих людях
Да, я срубаю бабло, но не на незнающих людях а на тех, кто не хочет знать. На пример на всяких тупых пятиклассниках, которых мамка приводит, чтобы я им делал домашку и объяснял школьную программу, на которую им срать. Ему объясняешь, а он ковыряется в носу, вытаскивает козюлю, рассматривает ее и жрет. Да-да, именно жрет. Есть и умные люди, которым я только объясняю и даю задания, а они все делают сами дома.
Все, я съебываю отсюдова.
Лингвист-кун

Аноним Вск 08 Июн 2014 00:53:02  #242 №201076 

>>201073

> ты ведь не можешь грамотно поддержать диалог или высказать грамотно мнение по поводу чего-либо

Еще как могу. Я на форчане нагибаю рачком таких как ты не хуже, чем тут.

> печальные истории про школьников

Лол, я таки попал, назвав тебя репетитором. А ведь как кличет себя гордо: "лингвист".

> но не на незнающих людях а на тех, кто не хочет знать
> Есть и умные люди, которым я только объясняю и даю задания, а они все делают сами дома.

То есть с умных людей ты денег не берешь? Или таки ты просто сам запутался в своей пиздоболии.

Аноним Вск 08 Июн 2014 01:11:55  #243 №201077 

>>201076
Учусь на лингвиста. В моей мухосрани сложно найти работу переводчиком, а переезжать не хочу.
>с умных людей ты денег не берешь?
Умные люди платят и получают, то что хотят.
Теперь точно съебываю.

Аноним Вск 08 Июн 2014 01:12:38  #244 №201078 

на самом деле самому учить конечно заебца, но как ты поймешь как произносить
the what three
и прочую хуиту
скорее всего если будешь изучать сам, ты даже не заметишь никаких проблем в своем произношении, только если на них кто-то укажет
мимопробегал

Аноним Вск 08 Июн 2014 07:41:40  #245 №201090 

>>201056
Кек, блядь, лол.
Я правильно понимаю, что ему не нравится, что число пи нацело пополам не делится, а нарисованный круг "нацело" пополам разделить можно?

Аноним Вск 08 Июн 2014 08:14:47  #246 №201092 

О, даже целый сайт есть с таким
http://stob2.narod.ru/

Аноним Вск 08 Июн 2014 08:37:49  #247 №201095 

>>201057
Видимо, он имел ввиду ординал.

Аноним Вск 08 Июн 2014 08:52:25  #248 №201096 

>>198318
Если проверить все камни разом, то выручка будет -1.
Если проверять по одному камню, то выручка будет -1.
Разобьём всё множество камней на группы по 32. Будет 31 группа из 32 камней и одна группа из 8 камней. Если радиоактивный камень окажется в одной из групп первого класса, то выручка будет 938. Если радиоактивный камень будет в группе второго класса, то выручка будет 961.
По теореме Ролля решение существует.

Аноним Вск 08 Июн 2014 10:33:35  #249 №201104 

>>201096
>По теореме Ролля решение существует.
Наркоман.

Аноним Вск 08 Июн 2014 10:51:50  #250 №201105 

Нужен учебник, где доходчиво объясняется теория, доказательство теорем по мат. анализу. Интересует интегральное и дифференциальное исчесление функции векторного аргумента.

Аноним Вск 08 Июн 2014 10:54:23  #251 №201107 
[url]

Видали? Какой-то поехавший мужик, который запилил кучу сайтов с рекламой своих репетиторских услуг. Пишет себе комментарии от лица других людей, создаёт видео от лица других людей (например, видеорелейтед). Короче, долго можно рассказывать, вы лучше сами взгляните.
http://online-tutor-in-moscow.blogspot.ru/
http://www.youtube.com/watch?v=Z0MseueSz3I

Аноним Вск 08 Июн 2014 11:01:19  #252 №201108 

>>201104
Почему?

Аноним Вск 08 Июн 2014 11:20:55  #253 №201109 

>>201108
Ролля можно было бы применять для функции, имеющей определенный вид, но у нас функция построена исходя из условий, которые меняются в процессе.
Вот этот анон пояснил суть задачи >>19981

Аноним Вск 08 Июн 2014 11:25:49  #254 №201110 

Хотел купить PS4, но в последний момент передумал.
Вот думаю, как потратить деньги на саморазвитие себя как математика. И что-то ничего в голову не приходит. Не блокноты-ручки же покупать.

Аноним Вск 08 Июн 2014 11:34:41  #255 №201111 

>>201110
Поезжай в магазин МЦНМО и купи там книжек.
Ректор университета просмотрел смету, которую ему принес декан физфака, и, вздохнув, сказал:
— Почему это физики всегда требуют такое дорогое оборудование? Вот, например, математики просят лишь деньги на бумагу, карандаши и ластики.
И, подумав, добавил:
— А философы, те еще лучше. Им даже ластики не нужны.

Аноним Вск 08 Июн 2014 12:00:37  #256 №201115 

>>201111
Книжки это хорошо, но я для них использую планшет.

Следующий Аноним Вск 08 Июн 2014 12:07:57  #257 №201116 
1402214877254.jpg

Какова вероятность, рандомно тыкнув в отрезок от нуля до единицы, попасть в рациональное число? А какова вероятность ткнуть в иррациональное число?

И сразу ответ:

И в том, и в другом случае -- ноль.

> Попасть в число - не невозможное событие. Но его вероятность нулевая

Аноним Вск 08 Июн 2014 12:15:42  #258 №201117 

>>201116
Вероятность попасть в рациональное равно отношению мощности счетного множества к континууму!

Аноним Вск 08 Июн 2014 12:58:02  #259 №201122 

>>201117
Конечно, но быдло постов 300 гадало-гадало, один порванный студик вспомнил сигма-алгебру и выдал, что вероятность попасть вообще в число, хоть какое, равна нулю, потому что точка имеет меру нуль.

Аноним Вск 08 Июн 2014 14:41:21  #260 №201130 

Анон, подкинь мне хорошие хранилища видео-лекций на английском.
MITшные нашел, но там как-то мало.

Аноним Вск 08 Июн 2014 15:27:22  #261 №201138 

>>201116
В иррациональное - 1.

Аноним Вск 08 Июн 2014 17:10:00  #262 №201145 

>>201138 --->>201122

Аноним Вск 08 Июн 2014 18:10:33  #263 №201151 

>>201095

А можно попроще, я адвансед числами не владею?

Аноним Вск 08 Июн 2014 18:11:54  #264 №201152 

>>201090
Похоже, что так.

Аноним Вск 08 Июн 2014 18:13:19  #265 №201154 

>>201115

Не деньги нужны для обучения, дурачок, а трудолюбие и ясность ума. Их за деньги не купишь.

Аноним Вск 08 Июн 2014 19:00:22  #266 №201158 

>>201154
Ты несомненно прав.


- Хотел научиться брать производную фреше, подскажите учебники по функану, аноны
- Не функан нужен для производной, дурачок, а тангенс наклона касательной. Для него функан не нужен.

Аноним Вск 08 Июн 2014 19:21:22  #267 №201160 

>>201151
Нет, я имел ввиду hereditary set

Аноним Вск 08 Июн 2014 19:30:22  #268 №201161 

>>201046

Что на счет обычного сета, вроде сета натуральных чисел? Он содержит нуль-сет?

Аноним Вск 08 Июн 2014 19:30:53  #269 №201162 

Повторяю свой реквест. Буду благодарен за годноту. >>201130

Аноним Вск 08 Июн 2014 19:39:51  #270 №201163 

>>201162
http://www.lektorium.tv/medialibrary
http://logic.pdmi.ras.ru/csclub/video
http://videolectures.net/

Аноним Вск 08 Июн 2014 20:23:20  #271 №201168 

>>201161
Видимо тот анон пытался описать подход к построению множеств в современной теории множеств. Идея за эти стоит примерно следующая:
1. имеется пустое множество,
2. если уже имеются какие-то множества, то из них можно скомпоновать новое множество.
Таким образом пустое множество лежит или в самом множестве или в каком-то его элементе или элементе элемента или элементе элемента элемента или ... .
>вроде сета натуральных чисел
В аксиоматической теории множеств обычно считают, что натуральное число 0 и пустое множество - это одно и тоже.

Аноним Вск 08 Июн 2014 21:17:58  #272 №201172 

>>201163
Ещё http://habrahabr.ru/post/141089/. И вообще Хабр полистай, там много полезного по этой теме.

Аноним Пнд 09 Июн 2014 00:20:43  #273 №201190 

>>198318
Лев в пустыне.тхт тред не читал

Аноним Пнд 09 Июн 2014 02:16:11  #274 №201202 

>>201168
Эм ... Или-таки множество, содержащее один элемент -- пустое множество?

Аноним Пнд 09 Июн 2014 03:46:56  #275 №201209 

>>201202
Я забыл здесь
>Таким образом пустое множество лежит или в самом множестве или в каком-то его элементе или элементе элемента или элементе элемента элемента или ... .
ещё одну альтернативу - множество может само быть пустым, т.е. не содержать ни одного элемента.

Аноним Пнд 09 Июн 2014 03:47:18  #276 №201210 

>>201202
Нет, пустое множество - это множество, не содержащее элементов.

Пустое множество является подмножеством любого множества, потому что не существует такого x, который бы принадлежал пустому множеству и не принадлежал данному множеству, следовательно, все элементы пустого множества принадлежат данному множеству.

Пустое множество 0 существует, это аксиома.
Для данного множества существует множество, содержащее его и только его, это аксиома.
Существуют множества 0, {0}, {{0}}, {{{0}}}, ...
Существует индуктивное множество, это аксиома.

Аноним Пнд 09 Июн 2014 09:01:49  #277 №201221 

>>201040
>Могут ли неупорядоченные множества содержать несколько одинаковых элементов? Ведь в противном случае и упорядоченные множества не смогут.
Тут прямо ряд фейлов. Математематика - это не ололо быдлокодерство. Множества не бывают упорядоченными (вернее бывают, но порядок определен отдельно, может существовать отдельно от множества, и внезапно - он тоже множество) или не упорядоченными. Это раз. Во вторых элементы не могут быть "одинаковыми". Одинаковые элементы - это один и тот же элемент. Это тебе не "объекты", которые ты горазд сравнивать. "Упорядоченные" множества можно строить как тебе хочется. Например множества векторов (тензоров), где второй координатой пары является индекс, тогда да. Можешь включать туда пары образованные из чта угодно + индекс. Можешь взять два своего что угодна с разными индексами и включить во множество. При этом надо осознавать, что это - разные элементы множества.

Аноним Пнд 09 Июн 2014 09:03:09  #278 №201222 

>Математематика
Вот это я выдал.

Аноним Пнд 09 Июн 2014 10:06:35  #279 №201230 

>>201221
Ты не сказал ещё про вполне упорядоченные множества, где нам не нужно явно указывать индекс каждого элемента, а достаточно сказать, что (i+1)-ый </> i-того.

Аноним Пнд 09 Июн 2014 11:08:41  #280 №201234 

>>201230
>про вполне упорядоченные множества, где нам не нужно явно указывать индекс каждого элемента, а достаточно сказать, что (i+1)-ый </> i-того.
Это называется последовательностью.
>Пусть задано некоторое множество X элементов произвольной природы. Всякое отображение f: N-> X множества натуральных чисел N в заданное множество X называется последовательностью
Заметил? последовательность - отображение.
А теперь:
>Отображение f:X -> Y есть множество пар (x,y) из X x Y, которое удовлетворяет... и.т.д.
Дошло?

Аноним Пнд 09 Июн 2014 11:23:09  #281 №201235 

>>201234
>Это называется последовательностью
А что насчёт континуума?

Аноним Пнд 09 Июн 2014 11:25:45  #282 №201236 

>>201210
Ты меня не так понял, я думал, что ноль определяется как {Ø}, а оказывается, просто Ø. Я забыл.
А не то, что Ø содержит один элемент.

Аноним Пнд 09 Июн 2014 11:28:12  #283 №201237 

>>201235
Он тоже является вполне упорядоченным.

Аноним Пнд 09 Июн 2014 11:30:55  #284 №201240 

>>201237
Конечно, но он-то не последовательность.

Аноним Пнд 09 Июн 2014 12:06:36  #285 №201242 

>>201240
Это фиксится с помощью бесконечных множеств индексов более высоких кардиналов, чем алеф-нуль.

Аноним Пнд 09 Июн 2014 19:22:59  #286 №201293 

>>201221

Я имел в виду упорядоченные как категория неупорядоченных. Например если определить упорядоченное множество (q, w, e, ...) как {q, {q, w}, {q, w, e}, ...}.

> Одинаковые элементы - это один и тот же элемент.

Я задал этот вопрос, потому что я не вижу применения классам эквивалентности. Если между каждым элементом всякого множества и каждым на нём классом существует инъекция, то зачем помещать квадратные скобки вокруг элементов?

> В аксиоматической теории множеств обычно считают, что натуральное число 0 и пустое множество - это одно и тоже.

Но не всегда допустимо из этого исходить. Порой требуется различать числа и конструкции нуль-сетов. Впрочем, читая пост ниже я таки нашел подтверждение тому, что подзабыл и был неуверен в:

> Пустое множество является подмножеством любого множества

> потому что не существует такого x, который бы принадлежал пустому множеству и не принадлежал данному множеству

Погоди, погоди. Это утверждение имплицирует, что нуль-сет содержит нуль-сет. Но это очевидно неверно. В случае если имеется в виду отсутствие такого х, которое принадлежит пустому множеству, следствие неверно.

Как мне подсказывает память, принадлежность нуль-сета всякому отличному от него самого сета является частью определения нуль-сета, то есть это договорная аксиома.

Аноним Пнд 09 Июн 2014 20:03:50  #287 №201297 

>>201293
Ты же не различаешь понятия принадлежности и включения. Принадлежать множеству и быть подмножеством множества - это совершенно различные вещи. Из того, что пустое множество содержится в любом множестве как подмножество, вовсе не следует, что оно принадлежит этому множеству в качестве элемента.

Аноним Втр 10 Июн 2014 05:55:42  #288 №201327 

>>201297

А, всё. Но ризонинг у тебя всё равно неверный.

Но тогда, конечно, возникает вопрос. Какой сет может иметь одноэлементное пересечение с каждым сетом семейства {{a}, {a, b}, {b}}?

Ах да, и сопутствующий вопрос, который забыл задать. А почему в учебнике Эндертона, например, аксиома выбора воспринимается как какой-то зашквар ("кое-где нам придется всё же использовать аксиому выбора, и в таких случаях мы об этом предупредим" - несколько преувеличил, но вы поняли, я думаю)?

Аноним Втр 10 Июн 2014 14:04:48  #289 №201347 

>>201293
>принадлежность нуль-сета всякому отличному от него самого сета является частью определения нуль-сета
Нет.
>то есть это договорная аксиома
Только в быдлокнигах.

Аноним Втр 10 Июн 2014 18:50:54  #290 №201389 

Известно, что не все учебники подходят для самостоятельного изучения математики (желательны ответы/разбор задач, соответствующая подача материала).
Кто знает годные для такого освоения книги по функ. анализу, диффгему и топологии?

От себя могу посоветовать "Теоремы и задачи функционального анализа".

Аноним Втр 10 Июн 2014 19:56:27  #291 №201393 

>>201327
Не понимаешь, почему аксиома выбора - зашквар?

Аноним Втр 10 Июн 2014 20:21:38  #292 №201396 

>>201393

Не понимаю. Не пойму только как она может быть истинной уже хотя бы в том примере, что я привел.

Аноним Втр 10 Июн 2014 20:27:03  #293 №201397 
1402417623276.png

Стал я повторять теорию множеств (напомнили) и столкнулся с неожиданным непониманием.

Вот мы взяли множество всех подмножеств, установили на нем отношение порядка по принципу "является начальным отрезком". Каким образом этот порядок может быть полным?
Например, во множестве натуральных чисел с обычным порядком подмножества
1. простых чисел
2. {25, 36, 49}
несравнимы.

Аноним Втр 10 Июн 2014 20:27:22  #294 №201398 

>>201327
Ты сперва с канторовской теорией множеств разберись, а потом уже лезь в аксиоматическую. Судя по постам выше, у тебя много пробелов и недопониманий на базовом уровне.

Аноним Втр 10 Июн 2014 20:38:22  #295 №201399 

>>201398

Позабывал многое, щито поделать десу. Но для аксиом-то адвансед знания не нужны же, они ведь аксиомы.

Аноним Втр 10 Июн 2014 20:47:08  #296 №201402 

>>201327
>Какой сет может иметь одноэлементное пересечение с каждым сетом семейства {{a}, {a, b}, {b}}?
В той формулировке, которую ты используешь, к данному множеству аксиома выбора неприменима. Множество должно быть расчлененным, то есть представлять собой семейство непересекающихся множеств. Пример такого множества: {{1,2,3}{4,5}{6}{9,8}}

Аноним Втр 10 Июн 2014 21:09:47  #297 №201404 

>>201397
>с обычным порядком
This.
Нужно брать не обычный порядок на N, а тот порядок, которым упорядочиваются подмножества из N: А начальный отрезок B, если А={a∈B|a<b}. Вот такой порядок будет полным.

Аноним Втр 10 Июн 2014 21:42:54  #298 №201407 

>>201404
Спасибо, анон, дошло

Аноним Втр 10 Июн 2014 22:44:40  #299 №201412 

>>198703
Без дополнительных условий это не верно. Пусть, например, мега-арбуз весит 60кг, а доступные гири имеют веса 20, 20, 20, 15, 15, 15, 15 и 12, 12, 12, 12, 12 кг.

Аноним Втр 10 Июн 2014 22:46:01  #300 №201413 

>>201412
Пардон. Глупость написал. Есть условие, что гирь только семь.

Аноним Втр 10 Июн 2014 22:51:18  #301 №201415 

>>198703

Думаю, тут нужно составлять систему нелинейных уравнений.

Аноним Втр 10 Июн 2014 23:05:36  #302 №201417 

>>198318

А можно намекнуть что примерно надо знать, чтобы эту задачу решить?

Аноним Втр 10 Июн 2014 23:29:55  #303 №201423 

>>201413
Пусть A={a1,a2,... a7} множество гирь, а A1, A2, A3 его подмножества, состоящие из 3, 4 и 5 элементов соответственно, такие, что сумма масс гирь в каждом из них равна массе арбуза. Ясно, что ни одно из этих подмножеств не содержится в другом, и A2∩A3, A1∩A3 не пусты.

По формуле включений-исключений, 7>=|A1∪A2∪A3|=|A1|+|A2|+|A3|-|A1∩A2|-|A1∩A3|-|A2∩A3|+|A1∩A2∩A3|. Отсюда следует |A1∩A2|+|A1∩A3|+|A2∩A3|>=5+|A1∩A2∩A3|.

Случай 1: |A1∩A2∩A3|>=1. Тогда |A1∩A2|+|A1∩A3|+|A2∩A3|>=6. Так как A2 не содержится в A3, то |A2∩A3|<=3, и следовательно |A1∩A2|>=2 или |A1∩A3|>=2 (или оба неравенства одновременно). Так как A1 не содержится ни в A2, ни в A3, то |A1∩A2|=2 или |A1∩A3|=2. Тогда единственная гиря в A1\A2 (соотв. A1\A3) имеет ту же массу, что и гири в A2\A1 (соотв. A3\A1),

Случай 2: |A1∩A2∩A3|=0. Имеем |A1∩A2|+|A1∩A3|+|A2∩A3|>=5, откуда следует, что хотя бы одно из этих слагаемых больше 2. Если это |A1∩A2| или |A1∩A3|, то действуем как в предыдущем случае. Если же |A1∩A2|<=1, |A1∩A3|<=1, то |A2∩A3|>=3 и, следовательно, |A2∩A3|=3. Тогда единственная гиря в A2\A3 имеет массу гирь из A3\A2.

Аноним Втр 10 Июн 2014 23:46:51  #304 №201428 

>>199714
Пусть e1(t),e2(t)...en(t) твой непрерывно меняющийся базис, который при t=0 был "правым". Запишем твои ei(t) в координатах этого базиса, помещая их в виде столбцов (или строк - неважно) в одну матрицу A(t) n-на-n. К примеру, A(0) будет единичной матрицей. Определитель det A(t) зависит от t непрерывно, и det A(0)=1>0. Следовательно, получить "левый" базис (для которого det A(t)<0), не допуская det A(t)=0, невозможно. Но если бы det A(t)=0 в какой-то момент t, то это бы означало, что e1(t),e2(t)...en(t) стали линейно зависимы, то есть перестали быть базисом.

Аноним Срд 11 Июн 2014 03:09:52  #305 №201461 

Верно данное утверждение?
http://www.youtube.com/watch?v=Mk6aNpI8LLY&feature=share&t=42m29s

Аноним Чтв 12 Июн 2014 13:00:11  #306 №201600 

Хуй знает чё вы тут изобретаете. Разделил бы камни на мелкие кучки и каждую бы проверил, кучка ссаных камней не стоит того мозгоёбства, что вы тут развели.

Аноним Чтв 12 Июн 2014 22:29:08  #307 №201659 
1402597748189.jpg

Простите, что вас отвлекаю. Помните того казахского математика, который предположительно решил проблему тысячелетия? Полгода прошло, а новостей никаких.

Аноним Чтв 12 Июн 2014 23:05:40  #308 №201661 

>>201659
>Полгода прошло, а новостей никаких.
Будто с самого начала было не ясно, что так и будет
Кстати, а как там тот китаец, заборол таки платонизм или нет?

Аноним Птн 13 Июн 2014 01:50:12  #309 №201662 

>>201661

А чем закончился тред, где аноны находили подвохи и слали эмейлы?

Аноним Птн 13 Июн 2014 09:35:02  #310 №201679 

>>201661
Охуеть, про него ещё помнят.

Аноним Птн 13 Июн 2014 09:50:05  #311 №201680 

>>201679
Помнят-помнят. Ты лучше ответь.

Аноним Птн 13 Июн 2014 10:15:12  #312 №201687 

>>201662
Он утонул. И эмейлы слали не аноны, а один человек. Я. И он мне потом писал, что подаёт статью в журнал. Сказал, что напишет, как результат, и поблагодарил за замечания. Прошло полгода с отправки статьи примерно, если она состоялась. Алсо, возможно, он что-то дополнил на арксиве, но я не смотрел

Аноним Птн 13 Июн 2014 10:26:59  #313 №201691 

>>201687

А по существу-то ответил?
То есть ты не знаешь, опубликовали его или нет? То есть если опубликовали, то значит, что с высокой вероятностью его замечания действительно резонны?
А в чем суть работы была, кстати? Так, в леймановских термах.

Аноним Птн 13 Июн 2014 10:29:02  #314 №201692 
1402640942661.jpg

>>201687
Т.е. конструктивисты таки соснули? Я так и знал.

Аноним Птн 13 Июн 2014 10:53:00  #315 №201695 

>>201687
>Он утонул
Я его кстати схоронил.

Аноним Птн 13 Июн 2014 10:58:44  #316 №201696 

>>201695
Поделись, будь бобр.

Аноним Птн 13 Июн 2014 11:13:35  #317 №201697 

>>201696
http://rghost.ru/56348468

Аноним Птн 13 Июн 2014 11:44:41  #318 №201701 

>>201697
Ах ты ж, сучёнок! Почти запустил ведь.
Вот за это и люблю хиккач - всегда приходится быть sharp

Аноним Птн 13 Июн 2014 12:06:20  #319 №201702 

>>201697
Спасибо.

Аноним Птн 13 Июн 2014 12:30:02  #320 №201704 

>>201701
>Почти запустил ведь.
Ты о чем вообще? MHTML никогда не видел штоле?

Аноним Птн 13 Июн 2014 13:13:04  #321 №201711 

>>201697

Эх, было же время. А ведь я их с тех пор зорича так и не прошел. Треклятый университет со своей рабораспорядительной иерархией образованцев.

Аноним Птн 13 Июн 2014 14:10:40  #322 №201717 

>>201711
Зорича ты не прошел не из- за универа, а из- за того, что ты ленивый хуй, не могущий в организацию своего времени.

Аноним Птн 13 Июн 2014 15:07:00  #323 №201725 

>>201692
Да ну, ты что? Ты говоришь как плебей. Конструктивисты всегда на коне. Причём тут китаёз?

>>201691
По существу у меня своих научных проблем было по уши, как и сейчас тащемта, не до того было. А по этой теме я переключился на одного голландца. Долго с ним переписывался. Он пилит "графические" основания, в которых якобы нет символов (грубо выражаясь). Даже что-то пилил в пайтоне. Могу скинуть инфу, если чё.

>>201697
Последний пост тут мой кстати.

Аноним Птн 13 Июн 2014 15:36:58  #324 №201729 

>>201725
>Последний пост тут мой кстати.
Ну там вроде что- то еще писали в треде, но я проебал это сохранить

Аноним Птн 13 Июн 2014 15:38:19  #325 №201730 

>>201729
Там под конец переводчик ворвался.

Аноним Птн 13 Июн 2014 16:16:23  #326 №201734 

>>198318
Бутылка с пробкой стоит 10 копеек, причем бутылка на
9 копеек дороже пробки. Сколько стоит бутылка без пробки, если автор задачки Арнольд?

Аноним Птн 13 Июн 2014 22:54:37  #327 №201781 
1402685677147.jpg

>>201734
Задолбали со своим Арнольдом.

Аноним Суб 14 Июн 2014 12:19:37  #328 №201866 

>>201734
9,5 коп.

Аноним Суб 14 Июн 2014 16:02:38  #329 №201888 

Ребята, а по какой литературе за лето можно освоить тривиум? http://ium.mccme.ru/f04/experimental.html

Аноним Суб 14 Июн 2014 16:30:33  #330 №201890 

>>201888
По платиновой очевидно.

Аноним Суб 14 Июн 2014 17:08:10  #331 №201892 

>>201890
Издеваешься, да?

Аноним Суб 14 Июн 2014 19:30:52  #332 №201896 

>>201888
Ты студент или школьник?

Так или иначе,

Алгебра - Курс алгебры (Винберг), Линейная алгебра (Кадомцев)
Топология и анализ - Лекции и задачи по топологии (Вербицкий), что-то из Зорича/Фихтенгольца
Теория чисел - Теория чисел (Нестеренко)

Аноним Вск 15 Июн 2014 13:19:54  #333 №201953 
1402823994217.png

Помогите, пожалуйста, невменяемому анону. Как такое решается? С меня что-нибудь.

Аноним Вск 15 Июн 2014 13:35:03  #334 №201958 

>>201953
Вычти и прибавь равное число, блеать, аля гауссово распределение.

Аноним Вск 15 Июн 2014 13:52:20  #335 №201961 
1402825940701.jpg

>>201958
Ох и вправду, жалко что я не умею считать. Держи прайслист.

Аноним Вск 15 Июн 2014 18:19:41  #336 №202017 

>>201953
Гауссово распределение симметрично относительно среднего значения - найти среднее минус а и среднее плюс а можно с равной вероятностью. Поэтому среднее значение должно лежать ровно посередине интервала (откуда и докуда значения вообще бывают). Чтобы пайти середину интервала надо сложить наименьшее с наибольшим и разделить на два.

Аноним Вск 15 Июн 2014 19:55:20  #337 №202031 

Аноны, помогите, с теорией вероятности знаком, но не очень понимаю, что требуется.

Предприниматель планирует выделить 10 у. е. на формирование порт-феля акций компаний К1, К2, К3 по ценам 3, 2, 5 у.е. соответственно. В конце года рынок ценных бумаг может оказаться в одном из состояний S1, S2, вероятности которых p и 1- р, соответственно. В таблице для каждого из состояний указаны ожидаемые дивиденды x и y . Возможные реше-ния: (3, 2, 5), (2, 2, 2, 2, 2), (5, 5), (3, 3, 2, 2). Рассматриваются критерии М и σ.
Требуется составить платежную матрицу доходов (стандартной формы), выделить Парето оптимальное множество решений и сформировать опти-мальный портфель акций используя взвешивающую формулу 2 М - σ.

Дивиденды, % К1 К2 К3 p
x 9 6 10 0,45
y 16 14 12

Аноним Вск 15 Июн 2014 19:56:57  #338 №202032 

>>202031
Блять, смазалось. Вероятность х 0.45, вероятность у 0.55,
дивиденды для х 9 6 10, лдя у 16 14 12

Аноним Вск 15 Июн 2014 21:14:55  #339 №202039 

>>201896
Школьник-кун.

Аноним Вск 15 Июн 2014 21:26:12  #340 №202041 

>>201896
Школьник-кун.

Пока что решаю первый листок.
Дико проёбался с экзаменами в универ.

Аноним Вск 15 Июн 2014 21:54:16  #341 №202042 

>>202041
За лето будет сложно все это освоить (если уходить дальше простейших формулировок), но реально. И, конечно, все зависит от твоего текущего уровня.

Аноним Вск 15 Июн 2014 22:40:31  #342 №202046 

>>202041
>Школьник
>проёбался с экзаменами в универ
Что-то я не понял... Абитуриент таки, а не школьник уже? Алсо зачем тебе вообще изучать всё это?

Аноним Вск 15 Июн 2014 22:48:21  #343 №202049 

>>202046
Это интересно, это раз. Ну и знания этого открываеть путь к знанию еще более няшных вещей.
Во вторых, математиком хочу же быть.

Аноним Вск 15 Июн 2014 22:49:57  #344 №202050 

>>202046
Это интересно, это раз. Ну и знания этого открываеть путь к знанию еще более няшных вещей.
Во вторых, математиком хочу же быть.
Да уже не школьник.

Аноним Вск 15 Июн 2014 23:03:56  #345 №202051 

>>202050
Имхо ты ступаешь на зыбкую почву, всерьёз считая, что тебе непременно нужно освоить программу Вербицкого, чтобы развиваться как математик. Если бы ты с детства занимался с хорошими преподавателями или несколько лет потратил на чтение серьёзной литературы и постоянные занятия, то это, возможно, было бы оправданно. Но раз ты спрашиваешь совета здесь, то вряд ли у тебя за спиной есть это всё. А значит, незачем соваться в такие дебри. Есть много хорошей литературы, по которой можно развиваться воннаби-математику-абитуриенту, например Курант-Роббинс, "Конкретная математика", целая библиотека свободных книг МЦНМО, всякие видеолекции (не НМУ, а что-нибудь попроще).

Аноним Вск 15 Июн 2014 23:13:46  #346 №202052 

>>202051
Тфю. Это же не програма Вербицкого, а первый семестр НМУ, в любом случае, я считаю, что лучший способ чему-то научится это пытаться одолеть то, что тебе едва-ли по силам. Винберга я знал до совета, о Нестеренко слышал, но как учебник не рассматривал. Единственное с чем ты меня познакомил это с топологией Вербицкого.

Я с математикой познакомился в шараге в колледже, где мне преподаватель сказала, что если мне так нравятся лекции, то мне надо было на мех-мат поступать.

Конкретная математика мне смахивает на научно-популярную литературу и такими книгами, как по мне, интерес не воспитать, ибо из радужных сказок, ты попадаешь в мир, где нет ничего из него. Интерес надо в себе воспитывать и интерес не научно-популярный, а настоящий.

Аноним Вск 15 Июн 2014 23:30:49  #347 №202057 

>>202052
>Я с математикой познакомился в шараге в колледже
Т.е. с математикой ты познакомился 2-3 года назад, явно с посредственным педагогом.
>Конкретная математика мне смахивает на научно-популярную литературу...
Ну-ну. Короче, раз такой умный, иди и изучай лекции с НМУ, а не сиди тут.

Аноним Вск 15 Июн 2014 23:37:18  #348 №202058 

>>202057
Конечно. В том, что у нас поголовно все посредственности - в этом ты прав безусловно.

>>Ну-ну. Короче, раз такой умный, иди и изучай лекции с НМУ, а не сиди тут.
Я собственно так и делаю, а вот переходить на личности совсем не этично. Сюда зашел может, кто и в правду дельный совет даст, а то собирать частями все это немного плохо.

Аноним Вск 15 Июн 2014 23:52:05  #349 №202060 

>>202058
>В том, что у нас поголовно все посредственности - в этом ты прав безусловно
Я не понял, сарказм ли это, но если сарказм, то вот мой ответ: раз ты сам назвал то место, где учился, шарагой, то и выводы я делаю соответственные. Я не говорил, что "у нас все посредственные".
>а вот переходить на личности совсем не этично
Если ты и вправду так хорош, что через пару лет после знакомства с математикой и обучения в шараге для тебя литература ниже уровня лекций из НМУ кажется совсем тривиальной, то никаких сарказмов от меня. А если очередной "я буду изучать программу НМУ"-школьник без фундаментальных знаний, то моё презрение легко понять.

Аноним Пнд 16 Июн 2014 00:06:02  #350 №202061 

>>202060
Это не сарказм, а горькая правда.
>>Если ты и вправду...
Я так и не говорил, кстати, похоже ты имел ввиду "Что такое математика", а не "Конкретная математика", ибо второе это Кнут, которую я, кстати, пытался читать в классе 8, забив после 30 страниц, но книга Куратна это и в правду научно-популярная литература.

О каких фундаметальных знаниях идет речь?

Аноним Пнд 16 Июн 2014 00:17:44  #351 №202062 

>>202031
бамп, помогите, очень надо

Аноним Пнд 16 Июн 2014 00:19:32  #352 №202063 

>>202061
Нет, я писал об обеих книгах.
То ты с математикой в колледже познакомился, то читал в 8 классе Кнута. Мне надоело с тобой говорить, пойду спать лучше.

Аноним Пнд 16 Июн 2014 00:24:42  #353 №202064 

>>202063
Я все время до этого интересовался же програмированием и на олимпиадках был и на различных кружках и на форуме советовали читать Кнута. Вот я прочитал его и забросил.

Кстати, как по мне, Кнут странный я его брал перечитывать.

Ну вот и что ты за человек? Переходишь на личности, не понимаешь, что чтения Кнута и знакомство с нормальной математикой это совсем разные вещи.

Аноним Пнд 16 Июн 2014 00:29:57  #354 №202065 

>>202064
И на вопросы не отвечаешь. Какие фундаментальные знания?

Аноним Пнд 16 Июн 2014 04:46:15  #355 №202068 

Сап, ночной. Кун, 18 лвл, 1 курс. Учу вышмат. Надо привести матрицу из квадратичного вида в канонический.
Буду рад любой помощи. Даже объяснению общего вида.
Квадратический вид - x1x2+x2x3

Аноним Пнд 16 Июн 2014 05:41:12  #356 №202069 

>>202052
>Единственное с чем ты меня познакомил это с топологией Вербицкого.
Тот анон не я.

>>202060
Я не совсем понимаю твоего презрения, мне хочется верить в человечество, что тривиум НМУ достигается за 3-6 месяцев любым выпускником, даже крайне далеким от матшкол. Конечно, не у всех хватит терпения (придется и школьную математику вспоминать в деталях), придется над "очевидными" мишыными сентенциями зависать по получасу-другому, но это другое, а презирать его за отсутствие фундаментальных знаний как-то нелогично.

>>202068
Метод Лагранжа, коэффициенты при квадратах нулевые, сл-но делаешь замены в духе
g1-g2 = x1
g1+g2 = x2

Аноним Пнд 16 Июн 2014 06:07:54  #357 №202071 

А че бинарный поиск не канает?
Делим по полам, 500 чистых продаем, делим по полам 250 чистых продаем и т.д. 10 делений (1000 чуть меньше 1024) и минус один "зараженный" камень и того 989р

Аноним Пнд 16 Июн 2014 09:25:36  #358 №202074 

>>202069
Спасибо. Извини, что перепутал. Просто так и есть. Может чуть дольше, может быстрее, но в принипе так и есть - главное это понимания и интерес, как по мне.

Аноним Пнд 16 Июн 2014 11:33:15  #359 №202079 

>>202071
проиграл с быдлокодеришки

Аноним Пнд 16 Июн 2014 11:59:46  #360 №202080 

Тред создавать не хочу, а здесь, вроде бы, близкая полемика. Поэтому спрошу здесь.
Как мне математически обозначить "множество всех последовательностей действительных чисел"
Я исхожу из того что ℝ✕ℝ или ℝ² это множество всех кортежей из двух чисел. Значит, если у меня чисел бесконечно, то я должен писать ℝ? Но знак бесконечности мне не нравится, может ℝℕ₀? или ℝcard ℕ₀?
Или вообще писать как множество всех функций ℕ->ℝ? А как тогда это писать?

Аноним Пнд 16 Июн 2014 13:59:32  #361 №202084 

>>202080
>может ℝ^ℕ₀
this
или просто словами

Аноним Пнд 16 Июн 2014 15:35:27  #362 №202095 
1402918527585.png

>>202080

Не слушай долбоёба сверху, вот как надо. Если писать в степени кардН, это будет множество всех бесконечных счетных последовательностей.

Если писать как множество всех функций, гораздо более громоздкая хуита выйдет и скорее всего обосрешься где-нибудь и выйдет некорректная запись. Конечно, преподу твоей шаражки похуй, хоть словами всё пиши.

Аноним Пнд 16 Июн 2014 15:39:09  #363 №202096 


А впрочем что это я, ты очевидно самообучаешься.

Да, кстати, как множество функций множество последовательностей не представить, ведь это будет пара натурального числа и последовательности, а не то, что нужно. Моя ошибка.

Аноним Пнд 16 Июн 2014 15:43:43  #364 №202097 

>>202096

Разве только если ты это заебашишь, как второкоординатную проекцию этого множества функций N -> R, но сам понимаешь.

И главное не забывай, что последовательность - это и функция с областью оперделения в натуральных числах, и упорядоченное множество с элементами в любом множестве.

Аноним Пнд 16 Июн 2014 17:23:09  #365 №202102 

>>202095
Поссал на петушка, не могущего в Латех.

Аноним Пнд 16 Июн 2014 17:36:28  #366 №202103 

>>202102

А нахуя мне латекс, если я самообученец? Нужды пока не было, вот и программы нет. Так что иди нахуй.

или двощ начал поддерживать его внезапно?

<math>&Omicron \prod - &xi &gamma $yotta </math>

Аноним Пнд 16 Июн 2014 17:39:11  #367 №202104 

ℝ✕ℝ

{{math|<VAR>&Omicron;</VAR>}}

Аноним Пнд 16 Июн 2014 18:29:28  #368 №202109 
1402928968336.png

Пикрелейтед. Как я мог догадаться представить всё в таком виде? Олимпиада для школьников СПбГУ, если кому интересно

Аноним Пнд 16 Июн 2014 18:57:07  #369 №202110 

>>202109

Если бы задротил методы решения олимпиадных задач. Не забывай, олимпиады они на то и олимпиады. Тестируют знания и умения примерно как айкью тестики тестируют ум.

Аноним Пнд 16 Июн 2014 19:23:52  #370 №202114 

>>202109
Усматриваешь паттерны различные в условии, пытаешься заменить что-то так, чтобы сократилось удобно, вспоминаешь свойства синусов-косинусов.

Пара догадок (которые в опытном мозге произойдут более-менее быстро) и на 5-6 варианте замен, которые придут в голову ты поймешь, что сможешь закончить задачу.

Аноним Пнд 16 Июн 2014 21:25:49  #371 №202120 

>>202110
>>202114
Короче говоря, я правильно полагал, что олимпиадки не сильно лучше ЕГЭ (для поступления в вуз). Т.е. всё равно придётся натаскиваться на типовые задачи и приёмы, и так же как и с ЕГЭ можно пролететь из-за банальной невнимательности.

Аноним Пнд 16 Июн 2014 21:48:14  #372 №202124 

>>202120
Нет, не совсем правильно. Твою задачу одаренный человек решит не потому, что он натаскан, а потому, что для него там все так же ясно, как для тебя доказательство теоремы пифагора.

Аноним Пнд 16 Июн 2014 21:53:32  #373 №202125 

>>202124
Таких "одарённых" единицы, а вот победителями и призёрами станут в основном те, кто учился во всяких матшколах, где его натаскивали на олимпиадные задачки.

Аноним Пнд 16 Июн 2014 22:14:53  #374 №202127 

>>202080
>множество всех последовательностей действительных чисел
Множество конечных последовательностей обычно обозначают
\mathbb{R}^*
Множество бесконечных -
\mathbb{R}^\omega (см. "Omega language").

Аноним Пнд 16 Июн 2014 22:21:57  #375 №202128 
[url]

>>202080
Карочи, вот тебе по хардоку, от БАГИНИ

Аноним Пнд 16 Июн 2014 22:48:40  #376 №202130 

>>202124
Ну вот честно говоря я не согласен. Тут надо банальное понимания взаимосвязи. Не важно, одарен ли кто-то или нет.

Аноним Пнд 16 Июн 2014 23:51:23  #377 №202136 
1402948283771.png

>>202109
Какое-то ебанутое решение. Если я ничего не напутал, то можно проще.

Аноним Втр 17 Июн 2014 01:17:23  #378 №202145 
1402953443948.png

>>202128
Какое-то говно непонятное. Лучше бы медленно объяснила что-то одно.

Аноним Втр 17 Июн 2014 04:48:09  #379 №202165 

>>202136
Как по мне, логичнее решение, молодца.

Аноним Втр 17 Июн 2014 04:54:41  #380 №202166 

>>202109
Решение какое-то говно, даже не читал. А задача очевидная. Сразу можно счиать, что ab=ax+by, увеличив x, если надо (неравенство ухудшится). Выражаем а, подставляем в неравенство и возводим в квадрат. Получаем неравенство о среднем после сокращений.

Аноним Втр 17 Июн 2014 07:41:45  #381 №202191 

>>202127

Ну и нахуя ты лезешь, тем более с неработающим латексом? Я уже всё ему объяснил, а ты лишний раз человека запутываешь, давая ненужную хуиту.
А всё из-за нарциссизма. Сволочь.

Аноним Втр 17 Июн 2014 07:54:16  #382 №202193 

млЯ я чет не понял я с бинарным поиском на вопрос ответил про макс выгоду или обосрался и идти обтекать?

Аноним Втр 17 Июн 2014 12:15:52  #383 №202209 

>>202193
Конечно обосрался, у тебя после первой проверки 500 камней станут радиоактивными. Правильное решение смотри где-то в начале треда.

Аноним Втр 17 Июн 2014 12:18:33  #384 №202210 

>>202191
ЛАТЕХ, а не латекс.

Аноним Втр 17 Июн 2014 14:13:47  #385 №202223 

>>198318
Объясните, как ряд с бесконечно убывающим общим членом может расходиться?

Аноним Втр 17 Июн 2014 15:43:45  #386 №202234 


Правильно ли я разумею, что формула вида F(X,Y) по определению может задавать лишь биекцию? То есть мы можем подставить что члены первого множества, что второго, и соответственно мы всякий раз будем получать сюръективную (потому что в область прибытия в итоге только элементы области значений попадут и никаких лишних) инъекцию.

Аноним Втр 17 Июн 2014 16:37:26  #387 №202243 

>>202223 -> >>200265

Аноним Втр 17 Июн 2014 17:42:57  #388 №202247 

>>202234
Нет не правильно. Во вторых таким обозначениям могут задавать не только функцию или отображению, но и еще высказательную функцию и отношения/etc/.

Аноним Втр 17 Июн 2014 17:50:15  #389 №202249 

>>202223
Общий член может убывать и стремиться к какому-то числу > 0.
Общий член будет всегда больше этого числа.

Аноним Втр 17 Июн 2014 18:19:02  #390 №202254 

>>202247

Но как тогда "извлечь" из такого выражения не биективное отображение?

Аноним Втр 17 Июн 2014 18:28:26  #391 №202258 

Элементы линейного пространства друг для друга не знают что такое умножение, так?

Аноним Втр 17 Июн 2014 18:37:46  #392 №202260 

>>202258
Нет

Скажи на милость, какой вуз?

Аноним Втр 17 Июн 2014 18:47:07  #393 №202262 

>>202260
Пока еще не поступил.
А как доказать, что в произвольном линейном пространстве нулевой элемент — единственный. Это все равно, что доказывать 2+2=4.

Аноним Втр 17 Июн 2014 18:55:14  #394 №202263 

>>202262
А я уж думал негодовать по поводу некачественных курсов ЛА у первоты. Ну а так ладно.

Ты не совсем ловишь суть: мы пытаемся задать что-то похожее на окружающий мир (пространство, объекты) с помощью минимальных логических утверждений (есть множество какой-то херни, о которой известно лишь то, что для любой другой херни можно сказать, принадлежит ли она этому множеству, есть поле чисел со своей аксиоматикой (на данном этапе можешь не заебываться и представлять просто R), и определены ТОЛЬКО операции с херней из множества и ТОЛЬКО по аксиомам).

Поэтому что-либо доказывать в духе аксиом пеано, распространяя их на множество нашей херни нельзя. Используй только аксиомы ЛП и иди от противного.

Аноним Втр 17 Июн 2014 19:07:16  #395 №202266 

>>202263
Понял, спасибо.

Аноним Втр 17 Июн 2014 21:33:39  #396 №202318 

>>202191
Стандартная нотация при обсуждении формальных языков как бы.

Аноним Втр 17 Июн 2014 21:50:10  #397 №202321 

Добоеб в треде. Хочу познать математику. Знания кончаются где-то на 7 классе да, полный пиздец Хочу во-первых к егэ подготовиться, во-вторых для себя знания, ибо стало очень интересна сама математика. Какие книги-учебники стоит читать для начала?

Аноним Втр 17 Июн 2014 21:59:44  #398 №202325 

>>202321
Книжки от МЦНМО (http://www.mccme.ru/free-books/, http://ilib.mccme.ru/). Например, всё так плохо, "Алгебра" Гельфанд/Шень, "Геометрия в задачах" Шень, "Простейшие примеры математических доказательств" Успенский, "Тригонометрия" Гельфанд/Львовский/Тоом.

Аноним Втр 17 Июн 2014 22:03:16  #399 №202328 

>>202321

Если ты на этот год егэ сдаешь, просто ботань тесты, не заботься о понимании. А потом уже выучишь.

Аноним Втр 17 Июн 2014 22:08:54  #400 №202331 

>>202325
Спасибо, иду обмазываться.
>>202328
Я не школьник, если что. Но таки да, ЕГЭ сдавать буду в 15 году.

Аноним Втр 17 Июн 2014 23:20:07  #401 №202343 

>>202254
Пусть X={1,2,3} Y={1,2}
F(X,Y) все.

Аноним Срд 18 Июн 2014 00:04:35  #402 №202351 

Подскажите учебник в стиле "физика для математиков". Сейчас вот сдал эту физику, но так и не понял до конца что к чему.

Аноним Срд 18 Июн 2014 13:11:30  #403 №202411 

>>202351

Я думаю любой приличный учебник сойдет. Просто на лекциях преподают самое важное от самого важного от самого важного и в итоге не то, что вся мат. строгие доказательства пропадают, ты даже всей физики не видишь, которая там есть. Хуйня для быдла короче, не гораздо лучше поп-книжек.

Аноним Срд 18 Июн 2014 14:07:59  #404 №202419 

>>202411
Я другой анон, но меня тоже интересует этот вопрос. Для меня, когда я учил физику проблемой было то, что регулярно встречались математически-некорректные рассуждения и построения. Это создавало ситуацию, когда я не мог двигаться вперёд так как рушилось понимание. При этом существенную часть курса я учил по учебнику, который был написан специально для математиков. Судя по тому, что я слышал о Ландау Лившице, там с этим ситуация ещё хуже.

Аноним Срд 18 Июн 2014 15:08:52  #405 №202424 

Что что, а вот дрочь на математическую строгость в учебнике по физике- совершенно бессмысленная вещь.

Аноним Срд 18 Июн 2014 15:33:12  #406 №202428 

>>202424
Дело даже не в особой строгости, а просто в ненаписании хуйни. Вот есть же арнольд по механике - и "физический смысол" не страдает и математика там - не просто блатной жаргон для записи охуенных приумок автора.

Аноним Срд 18 Июн 2014 15:49:14  #407 №202429 
1403092154604.png

Ещё раз большое спасибо, тем благородным донам, что помогли в прошлый раз.
Прошу помощи в последний раз. Поясните за ответы R2 и R3, как находится такая вероятность?

Аноним Срд 18 Июн 2014 16:21:52  #408 №202435 

>>202424
Это зависит от того, в чём цель учебника. Если цель состоит в том, чтобы познакомить математиков с некоторым разделом физики, то осмысленно. Так как усвоить строгое математическое построение математику будет гораздо легче, чем нестрогое. Правда видимо речь здесь не будет идти о формирование того же понимания, которым обладают физики. Но для последнего, судя по всему, требуется, чтобы математики стали мыслить, как физики - а это, по крайней мере, тяжело достижимо.

Аноним Срд 18 Июн 2014 16:55:13  #409 №202436 

>>202429
2)0,1 + 0,05
3)1 -(0,2 + 0,1 + 0,05)

Аноним Срд 18 Июн 2014 17:30:31  #410 №202437 

>>202436
Ах ты ученый!

Аноним Срд 18 Июн 2014 17:47:48  #411 №202438 

>>202436
Самое главное: спасибо!

Аноним Срд 18 Июн 2014 20:16:18  #412 №202443 

>>202435
>требуется, чтобы математики стали мыслить, как физики - а это, по крайней мере, тяжело достижимо.
Почему? Разве у нас вся актуальная физика не строится на строгой математике?

На ФФ МГУ физику тоже преподают довольно нестрого. Так что я, пожалуй, повторю реквест >>202351

Аноним Срд 18 Июн 2014 20:46:51  #413 №202450 

>>202419

Пробовал ли ты думать на счет того, что автором имелось в виду или задумывалось в том или ином шаге, и формулировать это корректно уже самому?

Аноним Срд 18 Июн 2014 21:00:38  #414 №202451 

>>202450
Математически корректно сформулировать многие рассуждения физиков нельзя, потому что часто нужный матаппарат ещё не создан. Более того, рассуждения физиков часто бывают неверны с точки зрения математики, и их нужно переформулировать. Скажем, Ландау на голой интуиции пренебрегал одними членами из разложения, но сохранял другие ввиду их некоей "физической значимости", рассуждения Ландау были непонятны потомкам до тех пор, пока математики не запилили теорию особенностей. Ссылки на некий физический смысл - это такой индикатор, обозначающий "хуй знает, как это логически обосновать, но наша интуиция говорит, что это должно быть вот так". Если требовать от физиков строгости, то они даже школьный курс не построят.

Подключайте libastral.so.

Аноним Срд 18 Июн 2014 22:11:07  #415 №202460 

>>202451
У тебя какая- то математика головного мозга.
В том и дело, что в физике главное не математическая строгость, а вот именно эта самая "физическая значимость" с "физическим смыслом", а то в результате мы получим очередные полтора землекопа. В физике в первую очередь нужно не "логически обосновывать", а не терять сцепление с реальным миром. И ссылка на физический смысл это такой индикатор, который обозначает что "это должно быть так, потому что иначе мы получим какую- то хуйню, противоречащую нашему опыту"
>Разве у нас вся актуальная физика не строится на строгой математике?
Весьма часто физика строится вначале на оче хуевой математике что кстати практически никак не сказывается на предсказательных и объяснительных способностях теории, которая уже потом допиливается.

Аноним Срд 18 Июн 2014 22:27:08  #416 №202465 

>>202451
>Более того, рассуждения физиков часто бывают неверны с точки зрения математики, и их нужно переформулировать
Главное, чтобы они были верны и подтверждались экспериментами, если это так, то точка зрения математики никого, кроме математиков, особо волновать не будет.

Аноним Срд 18 Июн 2014 22:42:18  #417 №202466 

>>202460
У физиков и у математиков приняты разные уровни строгости познания. Физик может что угодно объявить непрерывным и бесконечно-дифференцируемым, нимало не заботясь об обосновании, а математик знает, что далеко не все функции являются гладкими. Математика волнует вопрос, почему время моделируется именно вещественными числами, а не какой-то другой структурой, а физик о таких вещах просто не думает. Математика волнует понятие относительности и никому до этого нет дела, а физик просто говорит "в один и тот же момент времени" и нисколько не волнуется о смысле этих слов (когда несколько физиков всё-таки задумались, случилась известная революция).

>>202465
Они верны лишь в глазах физиков, ноне математиков. Такое было с обобщёнными функциями. Физики видели какой-то смысл в своём нелепом махании руками, а математики не понимали, что происходит. И с интегралами Фейнмана так же.

Аноним Срд 18 Июн 2014 23:11:13  #418 №202471 

>>202466
Физика имеет дело с реалом, а не с влажными абстрактными фантазиями. Соответственно "обоснование" в физике это совсем не то, что в математике. Ты же упорно пытаешься представить дело так, будто это что- то одинаковое, как для математики, так и для физики. Еще раз- в физике главное- это связь наблюдением и подтверждение в экспериментах, "математическая строгость"- второстепенна, просто потому что она НИНУЖНА в данном случае.
>когда несколько физиков всё-таки задумались, случилась известная революция
"Несколько физиков" все- таки задумывались о вполне конкретных вещах, с вполне конкретным физическим смыслом, не надо тут примазывать чистую математику.
>Они верны лишь в глазах физиков, ноне математиков
...мнение которых никого не волнует.

Аноним Срд 18 Июн 2014 23:17:40  #419 №202472 

>>202471
Если ты не можешь понять, что значит "фывапролдж", то ты не можешь заявлять, что эксперимент подтвердил "фывапролдж". Физики часто не понимают математики, которой пользуются.

Аноним Срд 18 Июн 2014 23:18:00  #420 №202473 

>>202471
>...мнение которых никого не волнует до тех пор пока они не научаться строить работоспособные физические теории исходя из чистой математики, без опоры на данные наблюдений и экспериментов.
Очевидный фикс.

Аноним Срд 18 Июн 2014 23:25:59  #421 №202476 

>>202472
>Физики часто не понимают математики, которой пользуются.
Да потому что для целей физики глубокое понимание математики не нужно, потому что это физика, а не математика и посредственное, с точки зрения математика, владение математикой никак особо не сказывается на работоспособности физических теорий. Ты же пытаешься подойти к физике так, будто это математика, а учебник по физике- учебник по математике и терпишь вполне неизбежный фейл.

Аноним Срд 18 Июн 2014 23:27:27  #422 №202477 

>>202476
А как ты узнаёшь, что теория работоспособна, если ты не понимаешь теорию?

Аноним Срд 18 Июн 2014 23:38:47  #423 №202481 

>>202476
>глубокое понимание математики не нужно
Кто там говорил про то, что чтобы быть физиком надо на отлично знать математику, на хорошо философию и на тройку с минусом собственно физику?
В любом случае, таки никто и не требует "глубоких" познаний. В оригинальном посте я просто просил учебник в котором хотя бы явной хуйни не написано. Например, я понимаю когда физики говорят "все функции гладкие" и не спорю, потому что это - соглашение. Но когда в каком-то учебнике я читаю охуительные истории например про отбрасывание "ненужных" членов по желанию левой пятки, то это не "физическое мышление" и не "достаточное для практики знание математики", это просто фактическая ошибка. По типу того что считать что 2+2=5 потому что лаборант пупкин это ПРОНАБЛЮДАЛ.

Аноним Срд 18 Июн 2014 23:49:55  #424 №202484 

>>202466

> Физики видели какой-то смысл
> а математики не понимали, что происходит

Конечно, потому что аутисты ёбаные. Уберомеги. Вся суть. Не доверяют собственному бессознательному и поэтому сводят всю жизнь к постоянным строгим сознательным проверкам. Физики, стремящиеся к математической строгости (почти все питарды) - более менее приспособившиеся, но всё еще омежки. И только настоящие альфы уверенны в себе и доверяют собственным бессознательным логическим процессам и в итоге получают колоссальные результаты. А омеганы как слыли в школе тормозами, так ими и остаются даже в академии, даже если достигают значительных результатов. Как Бор тот же.
Ньютон опирался на свою интуицию, создавая классическую физику и прекрасно работающий, несмотря на всю его интуитивность и нистрогасть, аппарат матана. Эйнштейн, Фейнман - все эти люди, пренебрегающие быдло антисоциальные уберменьши, бесстрашно шли по пути своего виденья к цели и никакие преграды в виде не сводящихся расчетов их не останавливали. А монотонную работу, которую может выполнить макака или робот традиционно поручали омежным клеркам-аутистикам, которые послушно формализуют новое открытие. Альфа создаёт машину, омега вносит эстетические модификации, чтобы утолить жажду своего ОКР.
Или вы думаете, что вот в школе вас травили, а вы потом такие дохуя элитки станете престижными учёными и будете вносить вклад? Ха! Академическими секретаршами будете в лучшем случае, на язык теории множеств ОТО переводить, лол.

Аноним Срд 18 Июн 2014 23:51:47  #425 №202485 

>>202481

Пиздец, вся суть. Пришел за учебниками и начал со взрослыми образованными людьми спорить. Подучись сначала, маня.

Аноним Срд 18 Июн 2014 23:54:03  #426 №202486 


>>202477

Она дает правильные прогнозы, блядь.

Аноним Срд 18 Июн 2014 23:54:43  #427 №202487 

>>202484
>и в итоге получают колоссальные результаты
Или начинают делить на вектор, что происходит гораздо чаще.

Аноним Срд 18 Июн 2014 23:55:27  #428 №202488 

> пренебрегающие быдло
> пренебрегающие быдлом

Селффикс.

Аноним Срд 18 Июн 2014 23:56:51  #429 №202489 

>>202487

Ну предоставь примеры, а то кукарекать от дымящегося пердака вы все горазды.

Аноним Чтв 19 Июн 2014 00:17:16  #430 №202492 

>>202484
>>202485
А что это ты, зелень, тут от имени "взрослых образованных" кукарекаешь? Со своим охуенным магическим пониманием физики и вообще природы. В школе вдохновили опытами про потирание палочки и переливание мочи из сосуда в сосуд?

Аноним Чтв 19 Июн 2014 08:20:40  #431 №202520 

>>202492
>магическим пониманием физики и вообще природы
Ты ебанутый?

Аноним Чтв 19 Июн 2014 08:37:27  #432 №202523 

>>202492
Ты пришел за ответами, решил вступить в дискуссию, а как хуями накормили семенить начал?

Аноним Чтв 19 Июн 2014 09:18:12  #433 №202524 

>>202484>>202485>>202523
Просто уйди от сюда со своими охуительными теориями. Такие, как ты не нужны.

Аноним Чтв 19 Июн 2014 09:24:59  #434 №202525 

>>202492
>магическим пониманием
Стоит отметить, что это свойственно в первую очередь именно математикам- увлечение всякими полумагическими, полурелигиозными системами понимания мира вроде Телемы или Каббалы, не говоря о том же платонизме как раз на почве чрезмерного логицизма и игнорирования "презренной действительности".

Аноним Чтв 19 Июн 2014 09:26:39  #435 №202526 

>>202524
Этот математик поломался, уносите.

Аноним Чтв 19 Июн 2014 11:28:03  #436 №202541 

>>202525
Ну вон этот товарищ наверху сам начал хуйню нести про "доверяй подсознательному кококо". Что до именно примитивного магизма, то он-то как раз математикам и прочим теоретикам несвойственен, им часто нравится религиозность и прочие теоретически проработанные учения, это да. Но я говорил про магизм в стиле "спляшем - пойдёт дождь", это как раз черта горе-экспериментаторов вроде экземпляра итт.

Аноним Чтв 19 Июн 2014 11:40:56  #437 №202542 

>>202525
лол, не стоит путать чувство глубокого охуевания от охуительности математики с игнорированием логики, являющейся вообще то её частью.

Аноним Чтв 19 Июн 2014 12:16:39  #438 №202549 

>>202541
>примитивный магизм
>теоретически проработанный магизм
Сорта говна

Аноним Чтв 19 Июн 2014 12:38:23  #439 №202551 

>>202549
Из теоретически проработанного в конце концов появились науки. Из примитивного появились суеверия и..более нихуя, разве что только медицина частично. Таки сорта.

Аноним Чтв 19 Июн 2014 13:46:41  #440 №202559 
1403171201914.gif

>>202542
>Логика - часть математики

Аноним Чтв 19 Июн 2014 14:08:49  #441 №202560 

>>202486
Если ты её не понимаешь, то ты не можешь утверждать, что вот это-то и это-то является прогнозом теории, потому что ты не можешь объяснить, каким образом это следует из теории. То есть если ты не понимаешь теорию, то и прогнозов у тебя нет. У физиков, как правило, есть только интуиция да видимость осмысленности. То, что они называют прогнозами, на самом деле есть только их охуительные прозрения.

Аноним Чтв 19 Июн 2014 14:35:15  #442 №202561 

>>202249
Окей, а почему тогда ряды с выполненным необходимым условием вообще могут сходиться, если общий член будет всегда больше нуля?

Аноним Чтв 19 Июн 2014 14:38:38  #443 №202562 

>>202560
>У физиков, как правило, есть только интуиция да видимость осмысленности.
а о самом их существовании ты знаешь только из гневных речей ацюковского. Нахуй иди.
>>202559
аргументируй что ли, а то детектирую у тебя МАТЕМАТИКА ЭТО НАУКА О ЧИСЛАХ

Аноним Чтв 19 Июн 2014 14:47:36  #444 №202564 

>>202561
А почему не могут-то? Сумма бесконечная, но и слагаемое бесконечно малое => сумма может получится конечной.

Аноним Чтв 19 Июн 2014 14:47:56  #445 №202565 

>>202561
Потому что всякая сходящаяся последовательность представима в виде ряда, всякий сходящийся ряд представляет некоторую сходящуюся последовательность. Рассмотри последовательность частичных сумм же. Например, последовательность 1, 2, 3, 4, 5, ... соответствует ряду 1, 1, 1, 1, 1, ...
Последовательность 0.1, 0.01, 0.001, ... соответствует ряду 0.1, -0.09, -0.009, ...
Последовательность 0.9, 0.99, 0.999, ... соответствует ряду 0.9, 0.09, 0.009, ...

Аноним Чтв 19 Июн 2014 14:49:32  #446 №202566 

>>202562
Сам иди нахуй. Я учился на физика, теперь переучиваюсь на математика, за слова отвечаю.

Аноним Чтв 19 Июн 2014 14:59:12  #447 №202568 

>>202566
>Я учился на физика
В Мухосранском заборостроительном техникуме?

Аноним Чтв 19 Июн 2014 14:59:57  #448 №202569 

>>202565
Что имеют в виду когда говорят что ряд сходится?

Аноним Чтв 19 Июн 2014 15:03:14  #449 №202570 

>>202569
Сходится последовательность его частичных сумм.

Аноним Чтв 19 Июн 2014 15:14:53  #450 №202571 

>>202570
Спасибо, всё прояснилось! И последний вопрос. Признак Даламбера не работает применительно к гармоническому ряду. Почему тогда его можно использовать без дополнительных проверок? И приведи пожалуйста ещё примеры рядов для которых выполняется необходимое условие, но они расходятся.

Аноним Чтв 19 Июн 2014 15:21:48  #451 №202572 

>>202571
Уточни, вопрос неясен. Хотя пофиг, я спать.

Ряд 1/2 + 1/3 + 1/4 + 1/5 + 1/6 +...
Общий член 1/n -> 0 при n->~
Последовательность частичных сумм 0.5, 0.8(3), 1.08(3), 1.28(3), 1.45, ...

Аноним Чтв 19 Июн 2014 15:29:20  #452 №202574 

>>202571
Он не выполняется для гармонического ряда.

Аноним Чтв 19 Июн 2014 15:32:53  #453 №202575 

>>202572
Ой, я сейчас посчитал, признак не даёт ответа, извини.

Аноним Чтв 19 Июн 2014 20:03:54  #454 №202615 

>>202524

А контраргументов так и не последовало. Предсказуемо прогнозируемо.

Аноним Чтв 19 Июн 2014 20:38:56  #455 №202622 

>>202615
Ты долбоёб? У тебя у самого ни одного аргумента, поехавший, просто высер из твоих охуительных фантазий.

другой-анон

Аноним Чтв 19 Июн 2014 20:47:47  #456 №202626 

>>202622
>другой-анон
Конечно, конечно, сёма, успокойся.

Аноним Чтв 19 Июн 2014 21:59:12  #457 №202642 

>>198318
по 500 камней. либо в ноль уйдешь либо шин.

Аноним Птн 20 Июн 2014 21:33:31  #458 №202739 

>>198318
Обозначим n число проверяемых камней в группе.
Вероятность, что в группе радиоактивный камень равна p = n * (1/1000) * (999/100)*^(n - 1)
Ожидаемая выручка:
g(n) = p*(-1 - n + 1000 - (1 +n)) + (1 - p)*n
Находим точку максимума функции g(n).

Аноним Птн 20 Июн 2014 21:44:47  #459 №202741 

Где почитать про уравнение Янга-Бакстера собственно применительно к стат.механике?

Аноним Птн 20 Июн 2014 21:53:36  #460 №202742 

Анон, человеческим языком поясни, что такое аналитическая функция?

Аноним Птн 20 Июн 2014 22:11:56  #461 №202747 

>>202742
Это такая функция, которая состоит из простых известных функций, которые легко рассчитать не прибегая к упрощениям.

Аноним Птн 20 Июн 2014 22:18:01  #462 №202748 

>>202747
Вернее будет сказать, что те функции которые ты описал являются аналитическими. Но это не все.

Аноним Птн 20 Июн 2014 22:36:08  #463 №202750 

>>202747
Ты сейчас описал элементарные функции. Я же спрашиваю, что такое аналитические функции кроме того, что они задаются ОЛОЛОФОРМУЛОЙ.

Аноним Птн 20 Июн 2014 23:02:13  #464 №202751 

>>202742
Функция комплексного переменного с определенными условиями на первые производные (Коши-Римана).Мысль в том, что функция "должна зависеть только от z". Практически, можно взять любую функцию f(x), разложить в ряд Тейлора и заменить x на z - получим аналитическую в некотором круге на комплексной плоскости функцию. Цимес в том, что существование всех производных кроме первой будет автоматически следовать.

Аноним Птн 20 Июн 2014 23:48:14  #465 №202755 

>>202751
Блестяще. Ассистент?

Аноним Суб 21 Июн 2014 05:31:45  #466 №202775 

>>202751
Огромное спасибо, теперь я точно всё понял.
-_-

Аноним Суб 21 Июн 2014 13:21:32  #467 №202794 

>>202775

Уёбывай отсюда, даун, он прекрасно всё объяснил. С такими смайликами на игрофорумчики пиздуй.

Аноним Суб 21 Июн 2014 14:37:37  #468 №202806 

>>202794
Ох, лол, все зачётики сдал уже, студент? Залатай уже жопу, я не виноват в том что вас в ваших баракахуниверах не научили отражать суть вещей без своего ненужного ебанутого мотиматического жаргона.

Аноним Суб 21 Июн 2014 17:33:38  #469 №202823 

>>202806
Дарагой, скажи тогда кто ты такой и чо те надо. Если ты читал ибанутый учебник комплексной переменной и ни фига не понял - это одно. А если ты хочешб чтобы тебе китайский разъясняли по-японски ,то это будет не четыре строчки а 20 страниц - и тебя все равно пошлют читать учебник. Два километра в один метр тебе никто не уложит.

Аноним Суб 21 Июн 2014 18:26:05  #470 №202827 

>>202806
>суть вещей
>Мысль в том, что функция "должна зависеть только от z".

Ты хоть не жопой читай.

Аноним Суб 21 Июн 2014 20:45:31  #471 №202838 

>>202827
И что это за ерунда? Возьми функцию действительной переменной, которая тождественный ноль на (-inf,0] и exp(1/x^2) на (0,+inf). Продолжение на комплексную плоскость не будет аналитической, она даже не дифференцируема, но "зависит только от z".

Аноним Суб 21 Июн 2014 20:51:37  #472 №202839 

>>202838
> exp(-1/x^2)

Аноним Суб 21 Июн 2014 21:49:53  #473 №202844 

>>202838
>>202838
"Зависит только от z" относится к интуитивному обоснованию нафиг нужны условия Коши-Римана и откуда они вообще берутся.
Ты можешь продолжить в комплексную плоскость экспоненту, заменив x на z - все будет пучком кроме точки 0, где будет особенность. Можешь продолжить ноль, который слева, получишь ноль во всей комплексной плоскости. Обе функции вместе ты продолжить не сможешь - у тебя нет приличного ряда Тейлора в нуле.

Аноним Суб 21 Июн 2014 22:04:31  #474 №202849 

>>202844
Ну возьмём вместо этой функции немного другую: ту, которая слева и справа exp(-1/x^2), а в нуле 0. Ряд тейлора в нуле по прежнему нулевой, но мы можем построить комплекснозначный аналог этой функции - exp(-1/z^2) если z=\=0 и 0 в нуле. Вот, пожалуйста, она зависит только от z, но не аналитическая.
Это я к тому, что непонятно что значит зависит только от z - |z| тоже зависит только от z например

Аноним Суб 21 Июн 2014 22:26:38  #475 №202854 

>>202849
Если брать exp(-1/z^2), то в нуле она просто не определена как аналитическая функция - у нее там дырка. В любой сколь угодно малой окрестности нуля ее болтает по модулю от нуля до бесконечности. Дополнять ее там нулем не имеет никакого смысла - она дополнится до плавной функции только если идти по вещественной оси. По чисто мнимой оси будет бесконечность.

"Зависимость от z" - скорее мнемоническое правило для Коши-Римана. Потому и в кавычках. Это не какое-то строгое утверждение.

Аноним Суб 21 Июн 2014 22:32:52  #476 №202855 

>>202849
Модуль z это корень из произведения z на z сопряженное - зависит в этом смысле не только от z.

Аноним Суб 21 Июн 2014 23:11:57  #477 №202856 

>>202854
>По чисто мнимой оси будет бесконечность.
Лолшто. Это же просто радиально-базисная функция exp(-1/(x^2+y^2)) от двух переменных. Ограничение её на любой прямой через нуль - это и есть одномерная функция действительного переменного, которую я и описал.

Аноним Вск 22 Июн 2014 00:13:17  #478 №202865 

>>202856
А вот хуй вам в грызло, уважаемый. z^2 это x^2-y^2+2ixy. 1/z^2 мне писать лень, но видно, что если z чисто мнимое, iy, то 1/z^2 будет таки -1/y^2, а вовсе не плюс. Отсюда и эрго...

Аноним Вск 22 Июн 2014 00:27:15  #479 №202868 

>>202865
А, лол, ну да, попутал, подразумевалось конечно |z|^2.

Аноним Вск 22 Июн 2014 00:36:13  #480 №202871 

>>202868
А раз модуль в квадрате, то это не аналитическая функция, потому как квадрат модуля это zz*. Аналитическая не должна зависеть от z*.

Аноним Вск 22 Июн 2014 01:08:35  #481 №202876 

>>202871
Ну да, в качестве примера не подходит. Но всё равно можно что-нибудь такое построить, зависящее от z но не аналитическое.

Аноним Вск 22 Июн 2014 02:49:38  #482 №202887 

>>202876
Из элементарных функций не построишь - так как все они пишутся через ряды Тейлора, то есть разумное продолжение в комплексную плоскость просто поставив z вместо х. Может быть неоднозначность.

Аноним Вск 22 Июн 2014 13:05:13  #483 №202918 

>>202887
Я просто пытаюсь уточнить "зависимость от z". Т.е. вот если мы подставим z=x+iy, то выражение функции можно переписать так, чтобы встречалось только x+iy. Например x,y, x^2+y^2, x-iy нельзя переписать в виде выражения от x+iy да? поэтому Re z, Im z, |z| и ~z не "зависят от z".

Аноним Вск 22 Июн 2014 14:20:07  #484 №202934 

Поцоны, помогите с пределом, второй день решаю.

lim(x->0) ln(x*ctg(x)) / (x*sqrt(1+2*x) - arcsin(x))

Аноним Вск 22 Июн 2014 14:55:52  #485 №202940 

Ребята, как вы занимаетесь соло? Т.е. без вузовских семинаров, самостоятельно? Как преодолеваете проблемы отсутствия решений задач, возможности быстро чекнуть верность понимания?

sageАноним Вск 22 Июн 2014 15:24:22  #486 №202946 

>>202940

Мы не безнадежные кретины, как ты, приятель.

Решение задач я делаю самостоятельно, а чекнуть в случае чего можно на миллионе интернет-ресурсов. Но вообще говоря в науке нужна уверенность в своём разуме. Как собственно и в жизни в целом. А иначе будешь склонен слушать других жуёбков и эмоции, вместо ума.

Аноним Вск 22 Июн 2014 16:00:06  #487 №202958 

>>202940
Если ты задаёшь подобные вопросы, то тебе стоит больше изучать математику очно. Так как математические построения как правило стройны и логичны, то если ты действительно хорошо понял достаточно большой кусок материала, то шансы, что понимание в корне ошибочно убывающе малы.

Аноним Вск 22 Июн 2014 16:22:32  #488 №202965 

>>202946
>>202958
Ясно, спасибо, анончики, значит проблема во мне. Видимо, слишком высокий для себя уровень взял.

Аноним Вск 22 Июн 2014 16:50:44  #489 №202969 

>>202918
Для этого и служит Коши-Риман. Пусть f(z) это u(x,y)+iv(x,y), и надо чтобы вся зависимость шла только в "правильной" комбинации x+iy. Тогда производная df/dx (буду писать частные как нормальные - значка нет на клаве) будет df/dz*dz/dx , то есть df/dz, а df/dy~df/dz*dz/dy~i*df/dz~i*df/dx. Подставляя через u,v в равенство первого с последним и приравнивая отдельно вещественные и мнимые части получим du/dx~dv/dy и du/dy~-dv/dx. Это и есть условие аналитичности. Легко проверить, что z^2 ему удовлетворяет, а x^2+y^2 - ни хуя - тут мнимой части вообще нет, а производные вещественной ненулевые.

Аноним Пнд 23 Июн 2014 01:46:40  #490 №203043 

>>202969
Да, всё верно.
Хороший учебник по анализу над комплексными желательно в алгебраическом изложении/связи с алгеброй можешь посоветовать?

Аноним Пнд 23 Июн 2014 02:21:18  #491 №203050 

>>203043
По-моему, Маркушевич нормальный. Я читал двухтомник, но у него вроде есть и чуть короче.Отсюда можешь скачать.

http://matan.far.ru/tfkp.htm

А то скачай оттуда все, и посмотри что тебе больше по стилю. Насчет связей с алгеброй - даже не знаю.

Аноним Втр 24 Июн 2014 15:09:06  #492 №203483 

/sci/ поясни за аксиому зависимого выбора, какие следствия, внятные её определения, чем ZF+DC в принципе лучше ZF+AC?

Аноним Втр 24 Июн 2014 15:33:18  #493 №203495 

>>203483

AC/DC, лол. Одна это постоянный ток, а другая - это переменный. )))

Аноним Втр 24 Июн 2014 16:04:56  #494 №203502 
1403611496850.jpg

>>203495
Пошел нахуй :3

Аноним Втр 24 Июн 2014 17:54:18  #495 №203525 

>>203483
Что то говно ...

Аноним Втр 24 Июн 2014 20:01:10  #496 №203543 

>>203483
Взглянул в википедию, там хорошо и достаточно лаконично написано http://en.wikipedia.org/wiki/Axiom_of_dependent_choice . Могу добавить, что это компромиссная форма выбора, которая довольно много позволяет доказать, но при этом совместна с рядом интересных аксиом, противоречащих аксиоме выбора (примером такой аксиомы, которая не упомянута в статье, является аксиома детерминированности AD).

Аноним Втр 24 Июн 2014 20:52:57  #497 №203563 

Можете по-быстрому объяснить безграммотному почему аксиома выбора считается зашкваром?

Аноним Втр 24 Июн 2014 21:02:47  #498 №203568 

>>203563
В основном людям не нравятся контринтуитивные следствия из неё, например теорема Банаха-Тарского. Ещё ряду конструктивистов она не нравится т.к. она неконструктивна, но это проистекает из их плохого понимания предмета - ZF ровно также неконструтивна, как и ZFC.

Аноним Втр 24 Июн 2014 21:29:17  #499 №203581 

>>203563
>считается зашкваром?
То есть?

Аноним Втр 24 Июн 2014 22:02:40  #500 №203590 

>>203568
> ZF ровно также неконструтивна, как и ZFC.
Тащемта, только разве что аксиома об индуктивном множестве

Аноним Втр 24 Июн 2014 22:08:14  #501 №203591 

>>203590
С аксиомой об индуктивном множестве, в смысле конструктивности, всё более-менее в порядке. Но вот об аксиомах выделения, замены и степени этого не скажешь.

Аноним Втр 24 Июн 2014 22:11:01  #502 №203593 

>>203581

Зорич пишет, что "породила горячие дискуссии в кругу специалистов", а Эндертон специально каждый раз отмечает, когда использует аксиому для получения теоремы. Ну и хиккачике читал уже неоднократно.

Аноним Срд 25 Июн 2014 06:46:42  #503 №203644 

Объясните, что такое бесконечность в стандартной алгебре. Википедия пишет:
>В математическом анализе, созданном на фундаменте исчисления бесконечно малых, вводится явно и абстракция бесконечно больших величин: ко множеству действительных чисел добавляются два символа +∞ и -∞ (строится расширенная числовая прямая R = {-∞} ∪ R ∪ {+∞}).
>Бесконечно большая — числовая функция или последовательность, которая стремится к бесконечности определённого знака.

Таки я не могу понять, бесконечность это число, как –6, например, или это нескончаемая последовательность чисел? Что значит выражение a∙∞? {a∙n1, a∙n2,…, nn,…}?

Аноним Срд 25 Июн 2014 10:31:03  #504 №203658 

>>203591
Но ведь без этой аксиомы нельзя определить вещественные числа двумя классическими методами.

Аноним Срд 25 Июн 2014 10:35:30  #505 №203659 

>>203644
Что за стандартная алгебра? Может, стандартный анализ?
Ты не путай extended reals со стандартным определением R.
В первых ∞ - обычная точка (проще всего себе представить "сворачивание" прямой в окружность или плоскости в сферу путём сшивания "точек на бесконечности"). Во вторых бесконечность - это не более, чем символ, но не число или последовательность. Последовательность может только стремиться к бексконечности.

Аноним Срд 25 Июн 2014 11:02:49  #506 №203662 

>>203659
>проще всего себе представить "сворачивание" прямой в окружность или плоскости в сферу путём сшивания "точек на бесконечности"
То есть -∞ < x < +∞?
> Во вторых бесконечность - это не более, чем символ, но не число или последовательность.
Не понятно. Что этот символ обозначает? Что такое ∞+a? Если это не число, то как складывается число и нечисло? Скажем, x → ∞ значит, что x растёт без ограничений.

Аноним Срд 25 Июн 2014 12:26:17  #507 №203685 

>>203662
>Что такое ∞+a
Да это укороченная запись, формально она не имеет смысла.
Сумма "∞+a" точно-формально должна записыватся как lim(x+a) при x->∞. Просто эта функция при x->∞ тоже стремится к бесконечности, что "неформально" записывается как "∞+a=∞".
Ещё можешь покопать в сторону т.н. нестандартного анализа - там напрямую постулируется существование бесконечно малых, и соответственно записи вроде "∞+a" имеют и формальный смысл.

Аноним Срд 25 Июн 2014 20:10:15  #508 №203782 

На счет бесконечно малых и больших. Они ведь, я так понимаю, и не постоянные, и не переменные. А есть ли название для этого типа объектов? Например, переянные или постоменные?

Аноним Срд 25 Июн 2014 21:23:33  #509 №203802 

>>203662
~, бесконечность в R, при теоретико-множественном подходе - это всего лишь элемент множества R, который отличается от всех других элементов этого множества. Связанное понятие - компактификация Александрова.
+~ и -~ - это два других множества, отличающихся от ~ и от всех остальных входящих в R множеств.

Аноним Срд 25 Июн 2014 21:26:13  #510 №203804 

>>203782
Ты какой-то ебанутый.

Аноним Срд 25 Июн 2014 22:43:46  #511 №203816 

>>203804

Да, вы, плебеи, издревле называете так всех, кто не подходит под ваши критерии "нормальности". Сколько гениев не прозывали ебанутыми за всё время.

Аноним Срд 25 Июн 2014 23:23:27  #512 №203823 

>>203816
Говна поешь.

Аноним Чтв 26 Июн 2014 11:57:20  #513 №203908 

>>203782
Они "постояннные" в том ключе в котором это слово вообще имеет смысл

Аноним Чтв 26 Июн 2014 13:07:26  #514 №203940 

>>203908

И в каком же ключе это слово имеет смысл? Я просто думал, что матанские бесконечности отличаются, скажем, от алефов принципиально тем, что "отчасти" являются числами. Понятно, что они всё равно отличаются. Где возможно про свойства разных бесконечностей прочитать?

Аноним Чтв 26 Июн 2014 14:04:40  #515 №203960 

>>203940
Алефы - это кардиналы, мощности множеств, классы эквивалентности равномощных множеств, как угодно.
"Бесконечности" в "матане" - неформальное описание расходимости чего-нибудь всё дальше и дальше.
Ты рассуждаешь как болтуны-философы, обращающиеся неаккуратно с собственным языком, то что и две разных вещи неформально называют "бесконечностями" ничего не значит.
В теории множеств "бесконечности" как мощности множеств бывают разные - счёт, континуум, гиперконтинуум итп.
В "матане" тоже бывают разные - различаются порядком роста, который важен, но не всегда.
Например, если тебе нужно найти предел f(x) при x->∞ тебе, в общем-то, всё равно что такое f(x) - x^2 или x например. Ответ в любом случае будет "∞". Но если у тебя
допустим f(x)=x^2/x или f(x)=x/x^2 то тут уже важен порядок роста, потому что в одном случае у тебя получится "∞/∞=∞" а в другом "∞/∞=0".

Аноним Чтв 26 Июн 2014 15:28:47  #516 №203982 

>>203960

Но бесконечность в матане ведь не может превышать континуум? Это по сути левелы внутри алеф один?

Аноним Чтв 26 Июн 2014 16:08:15  #517 №203990 
1403784495580.jpg

>>203982

Аноним Чтв 26 Июн 2014 20:08:29  #518 №204047 

>>203982
Ты ебанутый наглухо?

Аноним Чтв 26 Июн 2014 21:01:53  #519 №204056 

Объяснили бы лучше.
инбифо: биспалезна))

Аноним Чтв 26 Июн 2014 22:07:20  #520 №204072 

>>204056
Вот же >>203960 , алефы и бесконечности матана - не из одной оперы.

Аноним Чтв 26 Июн 2014 22:14:00  #521 №204074 

>>204072

Чем они будут являться тогда с точки зрения соответствующей параллельной теории (матан и теория сетов)? Или бесконечно малые или большие принципиально нельзя подать в множественно-теоретическом виде, т.е. они являются на данный момент не формализуемыми строго?

Аноним Чтв 26 Июн 2014 22:17:35  #522 №204075 

>>204074
>бесконечно малые или большие принципиально нельзя
Да можно смотри нестандартный анализ, уже писал. Причём тут мощности множеств только?

Аноним Чтв 26 Июн 2014 22:49:52  #523 №204086 

>>204074
ты ебанутый. Бесконечно малые что? Бесконечно малые последовательности, например, - это некоторые отображения из N в R.

Аноним Птн 27 Июн 2014 23:51:34  #524 №204300 

>>204086

Величины.

comments powered by Disqus

Отзывы и предложения